Quizzes day 1 through exam 2

अब Quizwiz के साथ अपने होमवर्क और परीक्षाओं को एस करें!

A patient presents to clinic and is noted to have a cataract in the right eye. What is an indication for urgent surgery?

Cataract induced glaucoma Lens- or cataract induced glaucoma is a secondary glaucoma in which the crystalline lens is involved in the mechanism of intraocular pressure (IOP) increase. The glaucoma may occur in open-angle or angle-closure forms, and requires urgent treatment to prevent vision-threatening complications.

Which of the following indications of increased ICP after head trauma appear first?

Restlessness and confusion The earliest symptom of elevated ICP is a change in mental status (especially irritability, and depressed level of alertness and attention). Cushing's triad (bradycardia, hypertension, and irregular respirations) occur later.

motor neurons embryonic source

Basal plate

The recommendations for a screening eye exam in a diabetic include:

At diagnosis in a 45 year man with type II diabetes

The medial motor system: (choose all that apply)

-Is responsible for posture and balance -Plays a role in orientation and "startle" reflexes It is helpful to lump tracts that control LMNs into a medial system and a lateral system based on the location of the tracts in the spinal cord. Medial system includes vestibulospinal, reticulospinal and anterior corticospinal tracts that run in the anterior white matter of the cord; these tracts predominantly target medially located motor neurons in the anterior horn, thus affecting axial (postural) muscle function, and balance. The lateral system (rubrospinal and lateral corticospinal tracts) is located in the lateral funiculus and targets motor neurons for limb movement. The corticospinal tract is the key tract for fine fractionated movement of the digits.

Tuberoinfundibular nucleus neurotransmitter

Dopamine

Accommodation for near vision is achieved by (select all that apply):

1. accommodation (lens rounding) 2. convergence 3. pupil constriction The lens contributes only about 30% of the optical power of the eye, but can adjust the eye's optical power. When viewing a distant object, you need weaker optical power, otherwise the image will be formed anterior to the retina. In this situation the lens is a flatter shape to reduce its optical power (it is said to be unaccommodated). For viewing a close object, the opposite occurs, and your lens becomes rounder (it is accommodated). These shape changes are accomplished by the ciliary muscle, a ring of smooth muscle encircling the lens. The ciliary muscle has little fingers that are attached to zonular fibers, which insert on the equator of the lens. Parasympathetic input to the ciliary muscle causes contraction of its muscle and relaxation of the zonular fibers and rounding up of the lens (accommodation). Accommodation is one component of a triad of actions that occurs in the accommodation reflex, when gaze shifts from a distant to a closer target. The triad consists of 1. accommodation (lens rounding) 2. convergence 3. pupil constriction

Match the letter

A = anterior median fissure B = base of the section; also the location of the corticospinal (pyramidal) tract C = CN XII (hint it exits between the pyramid and olive) D = surface bump overlying the inferior olivary nucleus E = nerve root that could be from CN IX or CN X F = solitary nucleus and tract G = tegmentum of medulla, also the position of nucleus ambiguus (the motor nucleus for pharynx and larynx)

Math the letter

A = cerebral aqueduct (another clue this is the midbrain) B = surface bump for colliculus (indicating this is a midbrain section) C = cerebral peduncle ie the base of the midbrain D = large nucleus that is black in vivo; supplies basal ganglia with dopamine E = nerve that emerges through interpeduncular fossa; important for pupillary light reflex and eye movement

In this CT scan, match the region with its function/description

A = executive function, higher cognitive function B = lobe of the brain that contains auditory cortex superior-laterally, as well as structures important for memory and learning inferior latterally C = the burried part of the cortex that processes taste and pain D = the part of the brain that if suffering a lesion would probably have defects in vision

In the coronal MRI, match the arrowed structure/space with its description.

A = insula B = putamen, deep to insula C = internal capsule, anterior limb (between caudate and putamen) D = caudate E = corpus callosum (large bundle of axons crossing the midline) F = anterior commissure (small bundle of axons crossing the midline) G = hypothalamus (part of diencephalon in the lower wall and floor of the third ventricle) H = medial temporal lobe (here specifically the amygdala)

Match with the structure and letter

A = internal carotid artery B = CN VI C = CN IX, X, XI exiting jugular foramen D = Vertebral artery

This is a myelin-stained section: myelinated tracts (white matter) stain dark, and nuclei (collections of neuron cell bodies) show up pale. The image is NOT in clinical image orientation. Match description with letter.

A = nucleus that makes the ipsilateral eye turn outwards (abduct) B = nucleus that contains motor neurons that innervate mimetic muscles C = largest white matter bundle that connects cerebellum and pons D = key descending tract in the pontine base This is a section through the lower half of the pons, containing the abducens and facial motor nuclei. You can see the fourth ventricle dorsally; the corticospinal and corticobulbar tracts coursing through the base of the section, surrounded by the pale-staining pontine gray (AKA pontine nuclei); and the impressive middle cerebellar peduncle (myelinated axons of pontine gray neurons projecting into the cerebellum),

Match the letter

A = olfactory epithelium B = retina C = taste bud in tongue epithelium D = Meissner's corpuscle in skin

Another myelin-stained brainstem section. Match description with letter.

A = tract and nucleus with role in taste and cardiorespiratory reflexes B = nucleus that gives rise to axons that leave brainstem between pyramid and olive C = nucleus in the lateral tegmentum, contributes axons to both CN IX and CN X D = nerve rootlet that exits lateral to the olive-could be part of CN IX or CN X E = axons forming the nerve that supplies motor innervation to the tongue F = axons of the descending motor tract, ie corticospinal tract

Match the arrowed structure with its description

A =Artery formed by union of vertebral arteries B = Nerve that carries sensory information from head and motor information from muscles of mastication This scan shows an axial view of through the mid pons: note its ventral smooth contour and the basilar artery located in the midline. Note also the fourth ventricle surrounded by the cerebellum posteriorly and pons anteriorly (CSF is bright in this image). Nerves that emerge in relationship to the pons are CN V shown here, and at the pons-medulla junction, CN VI (medial) and CN VII and CN VIII laterally at the cerebellopontine angle.

Damage to what part of the auditory pathway is the most common cause of sensorineural hearing loss?

Auditory hair cells. Sensorineural hearing loss is caused by damage to the neural part of the auditory system. Most commonly it is due to death of hair cells eg. from loud noises - gun shots, jack hammers, loud rock music, etc. Various drugs are also toxic to hair cells, including aminoglycoside antibiotics, and cisplatin, used in chemotherapy. You lose hair cells as you get older, starting with those sensitive to the highest frequencies. Another cause for sensorineural hearing loss is vestibular schwannoma.

REVIEW: blast from the past! You are evaluating the status of a patient who had a craniotomy 3 days ago. You suspect the patient is developing meningitis as a complication of surgery if the patient exhibits

A positive Brudzinski's sign Answer explanation: Signs of meningeal irritation compatible with meningitis include nuchal rigidity, positive Brudzinski's sign, and positive Kernig's sign. Nuchal rigidity is characterized by a stiff neck and soreness, which is especially noticeable when the neck is flexed. Kernig's sign is positive when the patient is supine, and with the thighs and knees flexed, the legs cannot be passively extended at the knees; a sign of meningeal irritation, specifically, is involuntary contraction of the hamstring muscles due to irritation of the nerve roots supplying them. Brudzinski's sign is positive when the patient flexes the hips and knees in response to gentle flexing of the head and neck onto the chest.

Match the structure

A= Corpus colosum B= Fornix C= Third ventricle D= Caudate E= Internal capsule, anterior limb F= Putamen G= Interior capsule, posterior limb H= Thalamus

Match the image with its likely scenario.

A=Athlete suffers blow to the head B=Elderly man falls and bumps head C=Patient complains of sudden headache worst of life, thunderclap etc. This is high yield information for boards etc! Note epidural hematoma at A with lens-shaped biconvex profile, subdural hematoma with crescent shaped profile (no blood can track down sulci) at B; and subarachnoid hemorrhage with blood pooling at base of brain at C -- notice how you can see the outline of the midbrain as the blood surrounds it flowing in the CSF in the basal cisterns of the subarachnoid space and extending into the lateral sulci.

Basal nucleus (of Meynert), septal nuclei neurotransmitter

ACH

Caudate and putamen (ie striatum) neurotransmitter

ACH

Motor neurons in cranial nerve nuclei neurotransmitter

ACH

Motor neurons in the PNS neurotransmitter

ACH

Neurotransmitter of the pre-ganglionic neurons in the ANS (both sympathetic and parasympathetic systems-review from Block 1)

ACH

A 39-year-old patient is evaluated after they sustained a penetrating injury to the right side of the neck. Physical examination shows they are unable to raise their right shoulder. Which of the following nerves most likely is injured?

Accessory Inability to raise the shoulder indicates a problem with trapezius. CN XI, the accessory nerve, supplies the sternocleidomastoid and trapezius muscles. CN XI arises from the cervical spinal cord and enters the intracranial cavity through the foramen magnum. It exits the skull along with CN IX and CN X through the jugular foramen, supplies motor innervation to the sternocleidmastoid muscle, then traverses the posterior triangle of the neck to reach the trapezius, to which it also delivers motor innervation. It is here that the nerve is vulnerable to trauma, eg. from penetrating wounds, or from surgery for lymph node biopsy, radical neck dissection, etc. The long thoracic nerve supplies serratus anterior: damage results in a winged scapula. The thoracodorsal nerve supplies latissimus dorsi: damage would cause weakness in shoulder extension, adduction and internal rotation. The suprascapular nerve supplies the supraspinatus and infraspinatus: damage results in weakness in abduction and external rotation of the arm.

Which of the following is the primary neurotransmitter used by lower motor neurons at the neuromuscular junction?

Acetylcholine Neuromuscular junctions are highly specialized sites of contact between LMNs and muscle fibers. These synapses use acetylcholine as excitatory neurotransmitter; the post-synaptic nicotinic acetylcholine receptors are ligand-gated cation channels that depolarize the muscle, thereby activating voltage-gated calcium channels. Succinylcholine is a neuromuscular blocker used to paralyze muscles during anesthesia (eg for endotracheal intubation); it is a non-competitive Ach nicotinic receptor agonist at neuromuscular end plate (depolarizing NM end plate) Rocuronium is similarly used, but it is a competitive Ach nicotinic receptor antagonist (selective for motor vs. autonomic nicotinic receptors).

The yellow splotches in this picture are drusen, which are deposits typical of

Age-related macular degeneration. AMD (age-related macular degeneration) is a degenerative disease affecting patients older than 55 years. Globally, it is the fourth leading cause of blindness. Drusen (yellow deposits of extracellular proteins) build up between the pigment epithelium and choroid. Drusen generally does not affect vision, but large deposits of soft drusen can eventually cause atrophy of pigment epithelial cells in the macular region (central 6 deg of retina), which leads to death of photoreceptors. So-called wet AMD also kills photoreceptors - in this case there is abnormal angiogenesis arising from the choroid, with small fragile vessels that rupture and eventually destroy neurons.

dorsal horn neurons embryonic source

Alar plate

Uncal herniation can result in which of the following? Mark all that apply.

Altered mental status Headache Ipsilateral pupil dilation Ipsilateral weakness Contralateral weakness All of these can occur due to uncal herniation. Any elevation in ICP can cause headache and altered mental state; compression of the midbrain in uncal herniation can interfere with the function of circuitry that regulates consciousness (ascending reticular activating system), leading to coma. Classically, as the uncus herniates over the tentorium it compresses the 1. ipsilateral cranial nerve III, causing ipsilateral pupil dilation (damaging parasympathetic fibers in it, and leaving the sympathetic innervation to dilator unopposed) and impaired pupillary light reflex. 2. the ipsilateral cerebral peduncle, in which the pyramidal (corticospinal) tract travels. This causes contralateral weakness because the pyramidal (corticospinal) tract decussates below this level (in the medulla). However, occasionally the displacement will be great enough to cause compression on both sides of the midbrain, leading to a combination of motor and pupil findings on each side.

The function of the ossicles in the middle ear is to

Amplify frequencies present in speech. The ossicles are the malleus, incus and stapes. The stapes is attached to the oval window. Vibration of the ossicles amplifies sound waves and conducts them to the base of the cochlear. Two small muscles, tensor tympani (innervated by trigeminal nerve and attaching to the malleus) and stapedius (innervated by the facial nerve and attaching to the stapes) limit the vibration, thus reducing the potential for damage from loud noise.

A skull fracture involving the squamous part of the temporal bone is most likely to result in which of the following?

An epidural hemorrhage A fracture of the squamous part of the temporal bone is most likely to cause an epidural hemorrhage, because the middle meningeal artery runs on the inner surface of the bone and can be damaged by bone fragments, for example, with head trauma such as a blow to the pterion. Damage to the middle meningeal artery leads to a build up of blood between the dura and the skull, creating a lens shaped, biconvex profile in imaging.

Following abdominal surgery, a 36 year old woman developed rapid onset bilateral hearing loss and vertigo. A history reveals that she had been treated for a post-operative infection. With a sinking feeling, you ask if she had:

Been treated with gentamicin or kanamycin. Hearing loss is classified as conductive hearing loss, which refers to any reduction in hearing that results from a non-neural problem of the auditory system, or sensorineural hearing loss, which is caused by damage to the neural part of the auditory system. This patient has sensorineural hearing loss. Hair cells are vulnerable to damage and death from various common insults. Most common are loud noises - gun shots, jack hammers, loud rock music, etc. The noise causes such violent shearing between tectorial and basilar membranes that the stereocilia are ripped off of hair cells. Various drugs are also toxic to hair cells, including aminoglycoside antibiotics (such as gentamicin and kanamicin), and cisplatin, used in chemotherapy. You lose hair cells as you get older, starting with those sensitive to the highest frequencies. Although they belong to the peripheral nervous system, hair cells do not regenerate, at least in humans (chickens are more fortunate).

A blockage of the anterior spinal artery at vertebral level T5 would most likely result in: (choose all that apply)

Bilateral Babinski's sign Bilateral loss of pain and temperature sensation from the legs The anterior spinal artery, a single midline artery, arises from the vertebral arteries in the skull, and runs inferiorly through the foramen magnum to supply the anterior 2/3rds of the cord, a territory encompassing the lateral corticospinal tracts (damage causes UMN type weakness and Babinski's sign), the motor neurons in the ventral horns (damage causes segmental LMN type weakness) and the anterolateral white matter of the cord where the spinothalamic tracts, which carry pain and temperature sensation, are located. The posterior part of the cord, where the tracts carrying proprioception are located, is supplied by the paired posterior spinal arteries. The long spinal arteries are supplemented along their length with blood from segmental artery branches. An enlarged branch of a segmental artery from the lower thoracic aorta (usually on the left), the great radicular artery of Adamkiewicz, is especially important in boosting blood supply to the cord. It can be damaged inadvertently in surgical repair of aortic aneurysms, leaving the patient paralyzed.

A vast array of systemically administered antibiotics and psychotropic medications are not able to efficiently enter the brain due to what intrinsic, anatomical brain characteristic?

Blood-brain barrier The blood-brain barrier is produced by tight junctions between capillary endothelial cells that serve to restrict permeability and reduce exchange of large molecules (such as antibiotics and psychotropic agents) between the blood and the brain cells.

The most likely localization of a lesion in a patient with sensory loss of the left face, arm and leg is:

Brain When sensory symptoms are on one side of the body and generally in a distribution that is larger than one sensory nerve or dermatome, think about the brain (specifically the cortex or underlying white matter, eg internal capsule) as the likely site of lesion.

The most likely localization of a lesion in a patient with crossed face and body sensory loss (ie loss of sensation on the face is on the other side from loss of sensation on the body) is:

Brainstem Crossed body and face symptoms generally localize to the brainstem. Because the long tracts in the brainstem largely target or originate in the opposite side of the body, while the cranial nerve nuclei and their axons supply ipsilateral face and head, a lesion on one side of the brainstem can manifest with sensory or motor symptoms on one side of the face/head and the opposite side of the body.

Which of the following would NOT be associated with a thrombosis in the stem of the right middle cerebral artery?

Broca's aphasia Language areas are in the vascular territory of the middle cerebral artery, but are present usually in the left cortex, hence a right MCA thrombosis would not affect language areas. The sensory and motor cortex for the face and upper limb are within the MCA territory and impairments in the contralateral face and upper limb would be expected with a thrombosis of the MCA.

Pick the dermatome that contains the smallest receptive fields:

C7 The smallest receptive fields would correlate with highest densities of sensory neuron innervation and regions of skin that have good two-point discrimination (eg finger tips and lips). You remember the dermatomes for the fingers from the musculoskeletal dermatome maps!

Fracture of the cribriform plate could result in damage to which of the following?

CN 1 Fracture of the cribriform plate can sever the axons of the olfactory receptor cells (bipolar cells) which run through the foramina in the plate to synapse in the olfactory bulb, resulting in anosmia. Olfactory neurons are a population of neurons that undergo neurogenesis, so the sense of smell may be restored over time. Extra pearl: if the fracture disrupts the dura and arachnoid of the anterior cranial fossa, CSF rhinorrhea can result (CSF leaks from subarachnoid space into nasal cavity) and needs to be managed to prevent spread of nasal bacteria intracranially.

Nuclei connected by the MLF (medial longitudinal fasciculus) include all the following EXCEPT

CN III, IV, VI and VIII but NOT cochlear part The MLF is a bundle of axons located close to the midline in the dorsal tegmentum of the brainstem; it connects several nuclei in the brainstem with each other, notably the vestibular and ocular motor nuclei, as well as brainstem nuclei with neck motor neurons. Connections through the MLF underlie the vestibulo-ocular reflex, voluntary conjugate eye movement, and normal coordinated head and eye movement. Damage to the MLF between the abducens nuclei in the pons and the oculomotor nuclei in the midbrain (as seen for example in MS) results in internuclear ophthalmoplegia.

A tumor at the jugular foramen (eg glomus jugulare), or, more rarely, a thrombosis at the jugular bulb (ie at junction between the sigmoid sinus and the internal jugular vein in the jugular foramen), can compress adjacent nerves and impair their function. The nerves that would be affected would include: (Select all that apply)

CN IX, X, XI Compromise of CN IX, X and XI results in problems with speech (hoarseness, breathiness, dysarthria), swallowing, shrugging shoulders, and turning the head; patients would have a decreased gag reflex, and an asymmetrical soft palate. Tumors in this region can extend to nearby CN XII, impacting tongue movements, and can grow upwards to involve CN VIII (affecting hearing and balance) and CN VII (causing facial weakness among other symptoms).

The arrows indicate the same foramina in CTs in axial (left) and coronal (right) planes. What structure traverses this space?

CN VII and CN VIII The space is the internal auditory meatus through which CN VII and CN VIII run. Notice how on each side, the IAC approaches the structures of the middle and inner ears.

Asymmetrical elevation of the palate and hoarseness indicate impairment of which cranial nerve?

CN X CN X (vagus nerve) contains motor fibers that innervate the muscles of the soft palate, pharynx and larynx, and is thus the main nerve involved in swallowing and phonation. Impairment of CN X can result in a hoarse voice (indicating paralysis of the vocal cord), a decreased gag reflex, and asymmetrical elevation of the soft palate when the patient is asked to say "Ahh". The soft palate muscles work together to elevate the palate in the midline; if CN X is damaged on one side, the soft palate muscles on the side of the damage are weak, so the palate will rise to the opposite side.

This imaging method is most commonly used to evaluate the possibility of acute brain hemorrhage

CT

In a playground accident, a 6 year old girl falls from a height of about 6 feet onto a hard surface, and loses consciousness. At the emergency room, she is still unconscious and is bleeding from the nose, but only minor facial abrasions are evident. Which imaging study would provide the most immediate and relevant information about the condition of this patient?

CT Head trauma and loss of consciousness raise a concern about skull fracture and the development of intracranial hemorrhage, which can raise intracranial pressure, leading to brain herniation and death. CT is fast, economical, and immediately provides information about life-threatening conditions such as skull fracture and/or acute blood around the brain. MRI is slower and may not be helpful in clearly identifying skull fracture and blood. Additionally, MRI can be frightening for a (conscious) young patient because of the noise. X-ray would only detect fractures, not bleeding. PET scan measures metabolic activity and little anatomical detail; it is not a standard testing procedure.

A patient presents with worsening headaches and transient obscurations of vision. On exam, bilateral disc edema is noted. The initial step in evaluation should be:

CT Head Bilateral disc edema (papilledema) raises concerns of increased intracranial pressure, which can be vision- and life-threatening. Pathogenesis Due to axoplasmic stasis caused to pressure on optic nerve from increased intracranial pressure. Can lead to optic atrophy; irreversible vision loss. Increased intracranial pressure leading to papilledema can be caused by A space occupying lesion (brain tumor) Decreased CSF drainage from obstructive hydrocephalus, Chiari malformation Decreased CSF absorption from meningitis, venous sinus thrombosis or outflow obstruction Idiopathic intracranial hypertension Evaluation Neuroimaging (either a CT or MRI) to rule out mass lesion prior to performing a lumbar puncture to evaluate CSF and measure opening pressure. Mass lesion must be ruled out prior to performing a lumbar puncture due to risk of herniation with a mass. Blood pressure should be measured as hypertensive emergency may present with bilateral disc edema and headache

Choose the correct order of the flow of aqueous humor through the eye.

Capillaries in ciliary processes across ciliary epithelium, to posterior chamber, through pupil to anterior chamber, to trabecular meshwork, to canal of Schlemm. Aqueous humor is produced in the posterior chamber by the ciliary epithelium (processes) and flows through the pupil to the anterior chamber to drain through the canal of Schlemm to the venous system. Problems in this system result in raised intraocular pressure and are associated with glaucoma.

A 72 year old man presents with ptosis and miosis of the left eye. Administration of cocaine and hydroxyamphetamine drops in the left eye do not change the pupil size. Possible causes of the man's symptoms may include:

Carotid artery dissection Ptosis and constricted pupil suggest impairment of the sympathetic innervation of the eye (Horner's syndrome). There are a number of places where damage can cause this: in the CNS, in the PNS affecting the preganglionic sympathetic neurons (from the upper thoracic spinal nerves to the sympathetic superior cervical chain ganglion), or in the PNS affecting the postganglionic sympathetic neurons, as they course in the carotid plexus and on branches of the carotid blood vessels to reach targets in the head. Recall that the neurotransmitter at the synapse between the pre and postganglionic sympathetic neurons is ACH, but the neurotransmitter at the terminal of the postganglionic neuron onto the smooth muscles is NE. Administration of eye drops in the clinic can help diagnose the lesion site: Hydroxyamphetamine releases norepinephrine from the nerve terminal and can be used to localize the site of a lesion in Horner syndrome. Patients with a central or preganglionic Horner syndrome will have an intact response to hydroxyamphetamine (the pupil will dilate) while a postganglionic lesion will not sustain dilation as there is no NE available Cocaine blocks reuptake of norepinephrine leading to pupillary dilation. If there is no available norepinephrine, the affected pupil will not dilate. Apical lung cancer can compress the sympathetic chain in the thorax/base of the neck, but would affect the preganglionic neurons. Postganglionic neurons in the carotid plexus receive their blood supply from microvessels of the carotid arteries; dissection disrupts this blood supply and the axon dies.

Some vestibular primary afferents bypass the vestibular nuclei and project to the

Cerebellum Vestibular connections to the cerebellum, whether directly from vestibular afferents, or conveyed by axons of the vestibular nuclei play an important role in balance. Vestibular inputs target the flocculonodular lobe and the medial parts of the cerebellum.

The tegmentum is the central core of the brainstem. A centrally located infarct in the tegmentum, within the reticular formation in the upper pons or midbrain, can result in which clinical presentation?

Coma or vegetative state The tegmentum of the brainstem contains numerous neurotransmitter-identified nuclei -eg raphe (serotonin), locus ceruleus (noradrenaline) and substantia nigra (dopamine), cranial nerve nuclei (eg nucleus ambiguus, solitary nucleus, oculomotor nucleus) as well as ascending (sensory) tracts (eg spinothalamic tract for pain and temperature and medial lemniscus for proprioception, vibration, pressure and discriminative touch sensation) and some descending (motor) tracts (eg corticospinal tracts). It also contains the reticular formation which coordinates many unconscious, homeostatic and reflexive processes including consciousness and arousal. An infarct or other damage (eg compression from a transtentorial herniation) to this reticular formation could potentially result in coma or vegetative state.

Which of the following is true regarding dorsal root ganglia (DRG) vs autonomic ganglia?

DRG contain the cell bodies of sensory neurons whereas the autonomic ganglia contain the synaptic junctions between preganglionic and postganglionic neurons. DRG and autonomic ganglia (eg. sympathetic chain ganglia, ciliary ganglion, otic ganglion, intramural ganglia) arise from the neural crest and are part of the PNS. DRG (and cranial nerve sensory ganglia, eg. trigeminal ganglion and geniculate ganglion) are collections of sensory neuron cell bodies with no synapses on them.

This imaging method is used to detect early ischemic changes

DWI

Timolol, a ß-adrenergic blocker, lowers intraocular pressure by:

Decreasing aqueous humor production Timolol lowers intraocular pressure by decreasing aqueous humor production. Recall that aqueous humor is secreted by the ciliary body in the posterior chamber, circulates through the pupil to reach the anterior chamber, and is drained through the trabecular meshwork and uveoscleral outflow pathways. Excess aqueous humor can raise intraocular pressure and lead to glaucoma. o Several different classes of pressure-lowering medications are available • prostaglandin analogues and cholinergic agonists increase outflow • ß-adrenergic blockers (such as timolol) and carbonic anhydrase inhibitors reduce production of aqueous humor • α-adrenergic agonists initially reduce aqueous humor production and subsequently increase outflow Adverse effects of timolol include ocular irritation and dry eyes, and its use is contraindicated in patients with asthma, COPD or bradycardia.

Left homonymous hemianopia can be caused by (pick all that apply)

Destruction of the right optic tract Destruction of the right visual cortex. Left homonymous hemianopia= loss of left visual field, which is seen by the right "sides" of each eye, ie left temporal retina and right nasal retina. Homonymous hemianopias can only occur with a post-chiasmatic lesion. Anterior to the chiasm, vision loss due to a single lesion would be restricted to the eye ipsilateral to the lesion. Lesions of crossing fibers at the chiasm cause heteronymous loss. specifically, bitemporal hemianopia.

Damage to CN XII on one side results in which of the following?

Deviation of the tongue ipsilaterally on protrusion CN XII (hypoglossal nerve) is a pure motor nerve innervating all the intrinsic and extrinsic muscles of the tongue, with the exception of palatoglossus. On protrusion, both sides of the tongue push against each other; if one side is weak, the tongue deviates to the weak side. The gag reflex tests CN IX (sensory limb) and CN X (motor limb) and the circuit in the medulla connecting them. The vocal cords are innervated by CN X. Damage to CN X results in paralysis of the ipsilateral vocal cord. Taste sensation on the tongue is carried by the facial nerve (anterior 2/3) and glossopharyngeal nerve (posterior 1/3).

A 48 yr old man presents with a movement disorder (chorea) and mental deterioration. MRI shows the loss of a structure in the lateral wall of the frontal (anterior) horn of the lateral ventricle. Which of the following is most likely lost in this patient?

Head of the caudate nucleus Lateral wall of the lateral ventricle is the location of the head of the caudate nucleus, which degenerates in Huntington disease. The ventricles are a good landmark for navigating inside the brain. The caudate lies in the lateral wall of the lateral ventricle; the hippocampus lies in the medial wall of the temporal horn of the lateral ventricle, after the ventricle has diverged from the midline and curved into the temporal lobe.

An arteriovenous malformation in the inferior division of the left middle cerebral artery would likely result in which of the following?

Fluent aphasia (Wernicke's) The inferior division of the MCA supplies structures in the temporal lobe. Of the options, only Wernicke's aphasia can result from damage to the temporal lobe.

Neurotransmitter of postganglionic neurons at most sympathetic terminals in PNS (review from Block 1)

Norepinephrine

A lesion of which of the following nerves would most likely have an effect on the gag reflex?

Glossopharyngeal The gag reflex is evoked by stimulating the back of the tongue or pharynx, and results in involuntary contraction of muscles of the larynx and pharynx. The sensory limb is CN IX and the motor limb is CN X, so the reflex is a quick test of the integrity of the glossopharyngeal and vagal nerves and the interneurons in the part of the brainstem that links them, ie the medulla.

Lumbar puncture in the case of suspected mass lesion (space-occupying lesion) should not be done without first acquiring neuroimaging, because of the risk of

Herniation of brain Intracranial masses can alter the disposition of the brain in the skull directly by compression and/or indirectly by compromising the amount, distribution and flow of CSF. Lumbar puncture in such cases can draw the brain downwards, resulting in compression of the medulla by the cerebellar tonsils, and life-threatening compromise of the medullary respiratory and cardiac control centers. Absolute contraindications for lumbar puncture are the presence of infected skin over the needle entry site and the presence of unequal pressures between the supratentorial and infratentorial compartments. The latter is usually inferred from the following characteristic findings on computed tomography (CT) of the brain: Midline shift Loss of suprachiasmatic and basilar cisterns Posterior fossa mass Loss of the superior cerebellar cistern Loss of the quadrigeminal plate cistern

A 70-year-old patient has a 2 day history of worsening generalized headache and increasing obtundation, and now complains of stiffness in the neck. On physical examination vital signs include T 38.7°C, pulse 85/minute, respirations 23/minute, and blood pressure 130/85 mm Hg. A CBC reveals a WBC count of 16,850/microliter. Serum electrolytes include a glucose of 88 mg/dL. A lumbar puncture yields cloudy cerebrospinal fluid with a glucose of 32 mg/dL, protein 146 mg/dL, and cell count of 3800 WBCs (95% PMNs and 5 % mononuclears) and 122 RBCs. The patient receives antibiotic therapy and improves. Which of the following long-term complications is most likely to develop from this individual's current disease?

Hydrocephalus This patient had bacterial meningitis, successfully treated with antibiotics. Meningitis affects the leptomeninges, ie. the pia and arachnoid. The scarring following the bacterial meningitis can lead to obstruction of CSF flow at the foramina of Luschka and Magendie resulting in obstructive non-communicating hydrocephalus, or alternatively scarring at the arachnoid villi/granulations can reduce CSF reabsorption, producing communicating hydrocephalus.

Two T1-weighted axial MRI scans of the brain of a patient with the P301S mutation of tau illustrate progression of cerebral atrophy over a 3-year period. The first MRI (A) was carried out when the patient was 37 years old. At that time point, the image is almost normal except that it shows mild cortical atrophy and ventricular dilatation that are advanced for the patient's age. The second MRI (B) was obtained when the patient was 40 years old. Note the severely progressed atrophy in the frontotemporal cortex and basal ganglia, especially profound in the caudate nucleus, with the corresponding marked ventricular dilatation. From the description and the images, the finding of enlarged ventricles illustrates an example of

Hydrocephalus ex vacuo Explanation: The patient has a neurodegenerative disease with cerebral atrophy: the loss of brain tissue is clearly visible on the right—the white matter is very thinned and gray matter deep in the brain is diminished. The expansion of ventricles 'fills the vacuum' left when tissue dies, so there is no elevation in intracranial pressure. Normal pressure hydrocephalus is typically characterized by urinary incontinence, dementia and gait problems, and is a condition seen mostly in elderly people.

Which of the following nerves does not carry the afferent component of a reflex?

III - oculomotor The oculomotor nerve is a pure motor nerve that carries motor fibers to the extraocular muscles (except superior oblique and lateral rectus) and parasympathetic fibers to the ciliary ganglion, which constrict the pupil and round up the lens. Thus CN III is involved in the efferent (motor) pathway of the pupillary light reflex and the accommodation reflex, but has no sensory role. The ophthalmic nerve carries sensation from the cornea - tested in the corneal blink reflex. The glossopharyngeal nerve carries sensation from the back of the tongue and pharynx - tested in the gag reflex.

Tumors originating from the Schwann cells of CN VIII (acoustic neuromas also known as vestibular schwannomas) compress CN VIII, causing ipsilateral sensorineural deafness, but they can also encroach upon adjacent structures. What functional deficits related to involvement of the cranial nerve closest to CN VIII might result? Pick all that apply.

Impaired taste sensation ipsilaterally Corneal dessication due to inability to close eyelid Drooling and loss of nasolabial fold ipsilaterally Increased ossicular chain movement CN VII attaches to the brainstem just medial to CN VIII, and exits the cranial cavity alongside CN VIII through the internal auditory meatus. A tumor on CN VIII can thus compress CN VII close to the brainstem, impairing its many functions.

An elderly patient with chronic otitis media (middle ear infection) might have all the following complications EXCEPT:

Inability to chew food due to injury to the mandibular division of the trigeminal nerve (CN V) Infections in the middle ear can affect the chorda tympani (taste from anterior tongue and parasympathetic supply to the submandibular and sublingual glands) as it traverses the space on the way to the infratemporal fossa tympanic plexus, causing pain and impairing the function of the lesser petrosal nerve (parasympathetic supply to the otic ganglion for parotid gland secretion). ossicles, impairing their mobility and function in transmitting sound waves The middle ear is connected to a number of spaces (INDE INDE INDE!!) including the mastoid air cells, and the Eustachian tube/pharynx; infections can spread to these sites.

The main role of myelin in the central nervous system is to:

Increase the speed of saltatory conduction

A lesion resulting in paralysis of all the facial muscles on one side of the face would be found where?

Ipsilateral facial nerve (CN VII) Paralysis of the entire hemi-face (as is seen in Bell palsy) usually signifies damage to CN VII (facial nerve) at some point along its peripheral course. Cranial nerves all innervate ipsilateral targets, so if the left CN VII is lesioned, the left face will be affected: the eye will not close properly, the forehead will be smooth, the nasolabial fold will not be apparent, the mouth will not close.

The epidural space:

Is a potential space located between the skull and the dura mater

Which of the following passes through the foramen magnum? Select all that apply.

Junction between brainstem and spinal cord Venous plexus Vertebral arteries The foramen magnum is the largest opening in the base of the skull. As well as the junction of the brainstem with the spinal cord, the vertebral arteries, anterior and posterior spinal arteries, CN XI, the internal vertebral venous plexus and the subarachnoid space all traverse the foramen. The sigmoid sinus does not leave the skull--it becomes the internal jugular vein at the jugular foramen. The internal carotid arteries enter the skull through the carotid canals. The hypoglossal nerves leave the posterior cranial fossa through the hypoglossal canals.

Which of the following supplies the inner ear?

Labyrinthine artery The labyrinthine artery from the basilar artery enters the internal acoustic meatus with CN VII and VIII, eventually supplying inner ear structures. Damage to it can cause ipsilateral deafness, though other causes of ipsilateral deafness are far more common (eg conduction deafness, vestibular schwannoma)

A 65-year-old right handed woman has sudden-onset word finding difficulties. Family members noticed that she has a difficult time stringing together more than 3 words and will often say only one to two word sentences. Surprisingly, her comprehension appears to be intact. On her exam, she is noted to have a subtle right lower facial droop as well as slight weakness in the right hand in addition to the language changes. Which of the following is the most likely localization of her stroke?

Left frontal lobe in the middle cerebral artery territory Because she is right-handed, her language center should be located in the left hemisphere (the majority of left-handed people also have their language centers in the left hemisphere). Her symptoms of right lower facial droop and right hand weakness localize to the left hemisphere, in particular, the left MCA distribution. Because her fluency is affected but with intact comprehension, this is most consistent with a Broca's aphasia. Her lesion is most likely in the left frontal lobe in the left MCA distribution.

Auditory hair cells are found in which location?

Location D A indicates the ossicles in the middle ear, B shows the vestibule (which is the location of the otolithic organs --utricle and saccule--that are part of the vestibular system), C indicates the vestibular part of the vestibulocochlear nerve (CN VIII) and D indicates the cochlea which houses the organ of Corti, the sensory apparatus for hearing, where the auditory hair cells are found. The vestibular system also uses hair cells as receptors-they are found in the crista ampullaris in each semicircular canal, and in the utricle and saccule.

A lesion in the lateral portion of the anterior horn of the lumbar spinal cord would result in:

Loss of motor control of the distal lower limb muscles The anterior horn is the location for LMNs, so damage here results in ipsilateral weakness, with decreased reflexes and tone, and eventual muscle atrophy. Within the anterior horn at the spinal cord enlargements, lower motor neurons are organized so that those innervating axial muscles are most medial, those innervating distal limb muscles (here, for toe movement) are most lateral, with motor neurons for intermediate muscles in between. Note that bladder motor neurons are not in the anterior horn. Bladder muscle is smooth muscle innervated by the parasympathetic system originating in the lateral horn of the sacral spinal cord; and the smooth muscle sphincter is innervated by the sympathetic system originating in the lateral horn of T12-L1 cord. Motor neurons for the pelvic floor muscles (skeletal) that you can contract voluntarily to control urination are in the anterior horn of the sacral cord (segments S2-4).

Lower motor neuron damage results in:

Lower motor neuron damage results in: This is another key point about the motor system. Weakness (loss of strength) can result from damage to UMN, LMN, NM junction, or muscle. UMN lesion: weakness, increased reflexes and increased tone (spasticity), no atrophy typically, Babinski with upgoing toes. LMN lesion: weakness, decreased reflexes and decreased tone, eventual atrophy, possibility of repair if axon is site of damage.

Vision loss in the setting of nonproliferative diabetic retinopathy is most commonly due to:

Macular edema Pathogenesis of diabetic retinopathy: Exposure to hyperglycemia over an extended period may cause endothelial damage in retinal vessels. Clinical features/presentation Diagnosis is made by slit-lamp examination and optical coherence tomography (OCT). Fluorescein angiography (injection of fluorescent dye into blood stream to allow visualization of blood vessels in vivo) shows retinal capillary leakage Classification of diabetic retinopathy is based on the presence of neovascularization. Nonproliferative DR is characterized by microaneurysms, areas of capillary nonperfusion, nerve fiber layer infarcts, intraretinal microvascular abnormalities (IRMA), retinal edema, hard exudates, intraretinal hemorrhages, arteriolar abnormalities, and dilation and beading of retinal veins. Vision loss results from macular edema, macular ischemia. Proliferative DR is characterized by proliferation of extraretinal neovascularization, Vision loss results as a consequence of the neovascularization through vitreous hemorrhage, tractional retinal detachment and neovascular glaucoma.

From the ____________ auditory information is relayed via the auditory radiations to the primary auditory cortex in the __________.

Medial geniculate nucleus; superior temporal lobe. The auditory pathway follows this sequence: hair cells -> CN VIII->cochlear nuclei. From the cochlear nuclei the path continues ipsi and contralaterally (hence it is binaural) in the lateral lemniscus to various targets such as the superior olivary nucleus, eventually synapsing in the nucleus of the inferior colliculus in the midbrain. From here the pathway reaches the medial geniculate nucleus and then continues on to primary auditory cortex in the transverse temporal gyrus. A mnemonic for the thalamic relay nucleus: M for medial geniculate, M for music, ie hearing.

Where do fibers in the ascending pathway for discriminative touch decussate (cross the midline)?

Medulla n the pathways carrying somatosensory information to the cortex for conscious awareness, the axon of the second neuron in the "3-neuron relay" is the one that crosses the midline. Thus for discriminative touch from the body, the axons of the dorsal column nuclei (ie gracile and cuneate nuclei) cross the midline in the medulla and ascend to the thalamus. In contrast, pain information from a part of the body crosses the midline in the spinal cord, close to the level of the spinal nerve carrying the information into the CNS. The axons of the dorsal horn nuclei on which the primary pain afferents synapse cross the midline in the anterior white commissure, and ascend to the thalamus.

As her new PCP, you are seeing a 37 yr old woman who complains of episodic spells of dizziness over the past few years. Her newly-retired PCP diagnosed and treated these episodes as vestibular neuritis, recommending bed rest, and treating with corticosteroids and anti-emetics. The patient states she has episodes of vertigo and tinnitus in her right ear a few times a year. The episodes "come out of the blue" and can last for 20 minutes or as much as 24 hours. During the attacks she experiences nausea and vomiting and has trouble with her balance and vision. An audiogram reveals low frequency hearing deficit in the right ear. What is the likely diagnosis?

Meniere's disease This patient demonstrates the classic triad of episodic vertigo, tinnitus and hearing loss that indicates she is likely suffering from Meniere's disease. The trouble with her vision during the attacks results from nystagmus induced by vestibular system dysfunction, so she has difficulty controlling eye movements, rather than specific visual system impairment. Vestibular neuritis presents with sudden onset vertigo, lasting 1-2 days, but patients typically do not have recurrent attacks. Vestibular schwannoma would cause the patient's symptoms but would not typically be episodic, and the hearing loss may be more dramatic. Psychosomatic disorders can sometimes involve hearing and balance symptoms, but would not typically show pathology on audiogram. Temporal bone fracture could disrupt the auditory and vestibular systems, but there is no history of trauma in this patient. from UpToDate: Meniere disease is a peripheral vestibular disorder attributed to excess endolymphatic fluid pressure, which causes episodic inner ear dysfunction. Affected patients present with spontaneous episodic vertigo lasting for minutes to hours, usually associated with unilateral tinnitus, hearing loss, and ear fullness. The vertigo associated with Meniere disease is often severe and associated with nausea and vomiting and disabling imbalance. The disequilibrium may last for several days. Horizontal-torsional nystagmus is typically seen on examination during an attack. The diagnosis of Meniere disease is suggested by the history. A low frequency sensorineural hearing loss on audiometry and a unilateral reduced vestibular response on electronystagmography help confirm the diagnosis. Exacerbations of Meniere disease may last for months or years during which time episodes can occur as frequently as every few days. The condition can go into remission spontaneously or with treatment, and it can recur.

In diabetic neuropathy, deteriorating vision is caused largely by

Microaneurysms of retinal blood vessels, resulting in patches of neuronal death. Diabetic neuropathy is currently the leading cause of blindness among Americans aged 20-64 years. It affects up to 80% of patients who have suffered from diabetes for at least 10 years. Microaneurysms of retinal vessels cause patches of neuronal death. In more advanced stages of the disease, new retinal blood vessels grow and, due to their fragility, commonly rupture. Hemorrhages cloud vision and take months or years to clear, or may never clear.

A 25-year-old man falls while parkouring falls (video) and strikes the left side of his head against a concrete retaining wall. On physical examination only a minor scalp abrasion is present at the site of the impact, with minimal bleeding that stops in 3 minutes. He is initially alert following this accident, but then became unconscious 30 minutes later. A head CT scan reveals a convex, lens-shaped area of hemorrhage centered over the left parietal region. These events are most likely to be associated with damage to which of the following parts of the intracranial vasculature?

Middle meningeal artery This is an epidural hematoma with a blow to the thin bone underlying the temple; fractures here damage the middle meningeal artery as it rises from the foramen spinosum to supply the meninges on the inner aspect of the calvaria. Arterial bleeding results in a rapid accumulation of blood in the potential epidural space between skull and dura. The blood expands this space, bounded by suture lines, forming a "convex, lens shaped" profile in imaging. The bleeding compresses the underlying brain and there is risk for herniation. Significant bleeds require urgent neurosurgical intervention with evacuation of the hemorrhage.

Following a fall from a ladder while painting, a 40 yr old person is taken to the ER. Initially, they lost consciousness, but revived a few minutes later. During examination, they are alert and oriented and complained of headache. A small scalp wound is sutured, and a CT scan of the head done (see image). Which of the following is the most appropriate clinical plan for this person?

Observe the patient, surgery at earliest opportunity. This person has suffered a significant epidural hemorrhage.The lesion is large and there is a slight shift in the midline as well as compression of the lateral ventricle, indicating elevated intracranial pressure. This type of lesion may present with initial loss of consciousness, followed by a "lucid" interval (patient is alert and oriented), potentially followed by signs/symptoms of herniation resulting in death, if the bleeding continues. In general, rapid surgical intervention to evacuate the blood is necessary when the lesion is as large as this one, as they are unlikely to resorb. The patient must be carefully monitored to assess further signs of raised ICP.

Blockade of this receptor is used in the treatment of motion sickness

Muscarinic receptors in the CNS Scopolamine a muscarinic antagonist is used in the treatment of motion sickness.

Activation of this receptor is used in the treatment of glaucoma.

Muscarinic receptors in the periphery In treatment for glaucoma, pilocarpine activates muscarinic receptors leading to increased aqueous humor outflow.

Cell bodies of somatosensory receptor neurons that are located in dorsal root ganglia include neurons that detect (choose all that apply)

Muscle contraction. Touch Vibration Painful stimuli The cell bodies of primary afferents carrying all modalities of somatosensation from the body (both visceral and somatic structures) are located in the dorsal root ganglia. They are pseudounipolar neurons. For the anterior head and face, primary afferent cell bodies are mostly in the trigeminal ganglion.

dorsal root ganglion cells embryonic source

Neural crest (dr)

enteric nervous system embryonic source

Neural crest (en)

Schwann cells embryonic source

Neural crest (sc)

sympathetic ganglia embryonic source

Neural crest (sg)

In the fovea, there is

No vision in low light conditions. The fovea is a small, specialized region of the retina which is characterized by the highest density of cone photoreceptors: it is responsible for high acuity and color vision, and functions in photopic (medium to high) light conditions.

A young child undergoes a CT scan for hydrocephalus. The scan shows enlargement of the lateral ventricles, but normal-sized 3rd and 4th ventricles. What type of hydrocephalus is this, and where is the problem likely to be anatomically?

Non-‐communicating; obstruction of the interventricular foramina (ie foramina of Monro)

Hair cells located near the base of the basilar membrane (ie near the oval window) respond specifically to high frequencies because

Only this region of the basilar membrane vibrates in response to high frequencies. The basilar membrane is five times wider at its apex than at its base (the base is the end closest to the oval window). The membrane is flexible at the apex, but becomes increasingly stiff approaching the base. Because of its changing width and compliance, different parts of it respond well to different frequencies. A high frequency causes pronounced oscillation at the base but not elsewhere; a middle frequency (say, 2000 Hz) causes oscillation in the middle; a low frequency oscillates the apex but not the base. Each hair cell responds best to a particular frequency (its "best frequency"). The hair cell's best frequency depends almost entirely on where it sits along the length of the basilar membrane.

This imaging method is used to evaluate local brain metabolism and receptor localization

PET

Red flag symptoms worrisome for orbital cellulitis include:

Pain with eye movement Exam findings in orbital cellulitis include fever, proptosis (protrusion of eyeball), chemosis (edema of conjunctiva), conjunctival injection, eyelid erythema and edema, upper eyelid ptosis, restriction of extraocular movement, and pain with eye movement. Decreased vision, color vision changes, visual field defects and pupillary abnormalities suggest an associated optic neuropathy.

A lesion in primary motor cortex on the wall of the interhemispheric fissure would result in which of the following?

Paralysis of the contralateral foot The motor 'homunculus" is organized so that the lower limb is on the medial surface, the trunk/hip on the upper lateral part of the precentral gyrus, the upper limb (with a large hand representation) inferior to that, and the mouth, lips and tongue in the precentral gyrus adjacent to the lateral sulcus. This somatotopic organization spans the arterial territories of the ACA (medial hemisphere) and the MCA (lateral part of motor and sensory cortex).

A lesion in the right lateral funiculus of the upper cervical spinal cord would result in which of the following?

Paralysis of the right arm and leg Most of the corticospinal tract decussates (crosses the midline) at the level of the lower medulla just rostral to the spinal cord to form the lateral corticospinal tract; this tract runs down in the lateral white matter to terminate on motor neurons in the anterior horn. Damage to the tract in the cervical cord produces UMN type deficits below the level of the lesion, because the tract at this level contains all the axons destined for all the motor neurons below that level. Damage to the tract in the thoracic cord would only cause deficits in the lower limb because the corticospinal tract axons that target the upper limb motor neurons leave the tract at C5-T1 cord levels.

Cavernous sinus pathology, such as thrombosis, or carotid aneurysm, can impact structures that pass through the cavernous sinus. So, which of the following could be affected? (Select all that apply)

Parasympathetic nerves causing pupil constriction Sympathetic nerves in the carotid plexus CN III, IV, V1, VI The cavernous sinus has a number of important structures in it. The internal carotid artery runs through it, and CN VI runs along the lateral aspect of the artery. CN III, IV and V1 all run in the lateral wall of the sinus. Both parasympathetic innervation (carried in CN III) and sympathetic innervation (carried as a plexus surrounding the internal carotid arteries) pass through the cavernous sinus en route to the eye.

Which of the following statements refer to the sympathetic nervous system?

Postganglionic axons travel with cutaneous branches of peripheral nerves to reach sweat glands, arrector pilae muscles and smooth muscles of blood vessels in the skin The postganglionic neuron cell bodies are found in the sympathetic chain ganglia or in the prevertebral (pre-aortic) ganglia Preganglionic fibers are generally short, postganglionic fibers are generally long

The most likely localization of a lesion in a patient with sensory loss in the thumb and lateral 2 fingers of the hand is:

Peripheral nerve Sensory symptoms that are on one side of the body and generally in a distribution that matches one sensory nerve or dermatome indicate a mononeuropathy, often from compression, trauma or neoplasm. The thumb and medial two fingers of the hand are supplied by the median nerve: thus sensory loss here can indicate a median nerve pathology, such as occurs when the nerve is compressed in carpal tunnel syndrome.

A 50 yr old woman is diagnosed with a condition that causes excruciating pain near her lower lip and chin. The involved nerve attaches to the brainstem at which level, and exits the cranial cavity via which foramen?

Pons/foramen ovale The woman is suffering from trigeminal neuralgia (tic douloureux). The pain is caused by aberrant firing in the trigeminal nerve, in this case the branches of CN V3 which supply the lower lip and chin. The trigeminal nerve is attached to the brainstem at the pons. CN V3 (mandibular nerve) exits the cranial cavity via the foramen ovale. CN V1 (ophthalmic nerve) exits the cranial cavity via the superior orbital fissure. CN V2 (maxillary nerve) exits the cranial cavity via the foramen rotundum.

t common cause of a rhegmatogenous retinal detachment is:

Posterior vitreous detachment Rhegmatogenous retinal detachment is caused by liquefied vitreous passing through a retinal break into the potential space between the neurosensory retina and the retinal pigment epithelium (RPE), most commonly in the setting of a posterior vitreous detachment.

Cochlear hair cells are depolarized by an influx of ____________, which is concentrated in the endolymph by the action of the _____________.

Potassium; stria vascularis.

Myopia can be caused either because the eye is too long or because the cornea is too strongly curved.

Problems with the eye's optics are common, but most can be corrected with glasses or contact lenses. In many people the optical power of the eye is either too strong or too weak. In myopia (excess optical power), the cornea is too strongly curved for the length of the eye, so images are formed anterior to the retina. (Myopia can also be described by saying that the eye is too long.) In hyperopia (insufficient optical power), the cornea's curvature is too flat for the length of the eye, so images that would be formed posterior to the retina if it did not get in the way. (Hyperopia can also be described by saying that the eye is too short.) Glasses or contact lenses can compensate for these conditions by adding or subtracting optical power. Astigmatism is a condition in which the corneal curvature is not perfectly spherical, but instead is somewhat elliptical or otherwise distorted. Customized glasses can usually correct for astigmatism.

You are reviewing the record of a child with increased ICP and note a report that the child exhibits signs of decerebrate posturing. On assessment of the child, you would expect which of the following if this type of posturing were present?

Pronation of the arms and rigid extension of both the arms and legs Decorticate ("no cortex") posturing (also known as flexor posturing) occurs with severe lesion at or above the midbrain and involves flexion of upper limbs and extension of lower limbs. Decerebrate ("no cerebrum") posturing (also known as extensor posturing) occurs with severe lesion in the brainstem below the midbrain and involves pronation and extension of upper limbs and extension of lower limbs. These postures are seen with severe brain damage associated with coma. They indicate damage to descending motor pathways, with some brainstem function intact. Extensor posturing may carry a worse prognosis. The "mnemonic" for these is: decorticate posture: flexed arms "point upwards" indicating a lesion higher in the neuraxis, ie midbrain and above decerebrate posture: extended arms point down to a lesion lower in the neuraxis: below midbrain.

If the dorsal columns on the left side of the spinal cord are cut, a patient loses which of the following? Mark all that apply.

Proprioception on the left side of the body. Fine (discriminative) touch on the left side of their body. The dorsal columns (gracile and cuneate fasciculi) are composed of the primary afferents for discriminative touch, proprioception, vibration and pressure. Hence, lesioning causes loss of these modalities. Because they are primary afferents, the loss is on the same side of the body as the lesion.

Axons coming from the nasal half of the right retina cross the midline to the left side of the brain because their

Receptive fields are located in the right visual hemifield. Three important principles to know here: 1. the lens inverts and reverses the image (so the nasal field is seen by the temporal retina for example) 2. each eye sees much of the opposite visual field in addition to the ipsilateral field, because of binocular overlap. 3. information from each visual hemifield is processed on the opposite side of the brain; the partial decussation at the chiasm allows this. The right visual hemifield (visual space to the right of a vertical line passing through the fixation point) is seen by both the nasal retina of the right eye and the temporal retina of the left eye. All right hemifield information is processed by the left side of the brain, so fibers from the right nasal retina cross at the chiasm to join the axons from the left temporal retina. Any post-chiasmatic lesion results in a contralateral field loss, and each eye will show some loss of vision when tested individually.

The corticospinal tract descends through all of the following locations EXCEPT:

Red nucleus The corticospinal (pyramidal) tract is comprised of axons of neurons in the motor cortex, that innervate motor neurons in the spinal cord. Some of these axons (ie those innervating the lowest motor neurons in the sacral cord) run the length of the neuraxis. The long course of the corticospinal tract makes it vulnerable to damage across a long stretch of the CNS, eg, the tract receives its blood supply from several different sources at different levels (eg MCA in forebrain, basilar artery in pons, anterior spinal artery in spinal cord) and a blockage in any one of them can cause irreversible damage.

A patient presenting with photopsias (flashes of light) should be seen urgently to evaluate for possible

Retinal tear Retinal tear or detachment refers to the detachment of the retina from its underlying support tissues. The photoreceptor layer detaches from the retinal pigment epithelium and choroid, disrupting the activity of retinal neurons. Retinal detachments are classified as: 1) rhegmatogenous, 2) tractional, 3) exudative. Rhegmatogenous (meaning arising from a fracture or rupture) retinal detachment is the most common type Presentation: Photopsias (flashes), floaters; curtain/shadow over the visual field. Caused by liquefied vitreous passing through a retinal break into the potential space between the neurosensory retina and the RPE, most commonly in the setting of a posterior vitreous detachment. Treatment: mainly surgical by closing retinal breaks by laser, cryopexy in addition to vitrectomy and/or placement of a scleral buckle.

Nissl stain reveals neuronal cell bodies due to a high density of:

Ribosomes within the rough ER Neurons actively synthesize proteins to maintain their long processes and to produce neurotransmitters. Their cell bodies therefore contain large amountsof rough ER. The ribosomes stain strongly with the classic histological Nissl stain and are sometimes referred to as Nissl bodies or Nissl substance.

A blockage of which of the following would most likely result in a Babinski's sign (with upgoing toes) and weakness of the left leg?

Right anterior cerebral artery Because the left leg is involved, and only cerebral arteries are listed, the location of a lesion must be in the contralateral cortex, hence the right. A lesion of the right anterior cerebral artery affects the sensorimotor cortex for the left leg. The posterior cerebral artery supplies the inferior surface of the temporal and occipital lobes and the medial surface of the cortex from the parieto-occipital sulcus posteriorly; damage to the PCA results in visual problems. It also supplies posterior thalamus. The MCA supplies the sensorimotor cortex for the face and upper limb, as well as part of the internal capsule (via deep penetrating branches). The anterior choroidal artery supplies choroid plexus in the lateral ventricles as well as part of the posterior limb of the internal capsule, where axons of motor cortex cells supplying upper and lower limb motor neurons travel.

The poor acuity of rods makes it impossible to read text by moonlight.

Rods can detect tiny amounts of light, but convergence in the subsequent pathway in the retina means that acuity is compromised.

A conductive hearing loss can be caused by

Rupture of the tympanic membrane. Conductive hearing loss refers to any reduction in hearing that results from a non-neural problem of the auditory system, eg blockage in the external auditory meatus, rupture of the tympanic membrane, damage to the ossicles (eg otosclerosis, a pathological bone growth of an ossicle, usually the stapes, which reduces the mobility of the ossicles), or damage to the middle ear (eg due to repeated ear infections or cholesteatoma).

Saccade movements of the eyes act to

Saccades bring new images of interest onto the fovea. The frontal eye field sends a specific movement signal to the superior colliculus and to saccade generators in pons and midbrain. Horizontal saccades are generated in pontine reticular formation. Vertical saccades are generated in midbrain reticular formation.

Repeated middle ear infections have destroyed the tympanic plexus in the middle ear cavity. The damage to preganglionic parasympathetic fibers that pass through the plexus would diminish production of:

Saliva by the parotid gland The preganglionic parasympathetic fibers of CN IX that form part of the tympanic plexus in the middle ear leave the tympanic cavity via the lesser petrosal nerve and synapse in the otic ganglion just inferior to the foramen ovale. From there postganglionic secretomotor fibers pass to the parotid gland by traveling with the auriculotemporal branch of CN V3. Mucous glands in the nasal cavity and soft palate, and the lacrimal gland in the orbit, are supplied by parasympathetic fibers in the greater petrosal branch of CN VII, that synapse in the pterygopalatine ganglion in the pterygopalatine fossa. From there, post-ganglionic branches travel along the nearest branches of CN V2 to reach their targets. Submandibular and sublingual salivary glands receive secretomotor innervation through the preganglionic parasympathetic fibers of the chorda tympani branch of CN VII, which join the lingual branch of CN V3 in the infratemporal fossa and synapse in the submandibular ganglion.

A delta and C fibers carry nociceptive information.

Sharp pain nociceptors (A delta fibers) are activated by intense mechanical stimulation and/or heat > 45 deg C. Dull pain nociceptors (C fibers) can be activated by substances released during inflammation or tissue damage (bradykinin, histamine, prostaglandins, etc.), by intense heat or intense cold, and by various noxious chemical substances (capsaicin in hot chili peppers, toxins in nettles, etc.)

A patient with severe conductive hearing loss in the right ear:

Should hear a tuning fork louder on the right in the Weber test. There are two tests that can be used to tell whether a patient has a conductive hearing loss as opposed to a sensorineural hearing loss: the Weber test and the Rinne test. The key in both tests is the conduction of sound, in this case vibration of a tuning fork, through the bone of the skull. In an ear that has a conductive hearing loss, bone conduction is unusually loud. Thus the patient will hear the tuning fork more loudly in her affected ear than in her normal ear, when the fork is placed on the center of the forehead (Weber). The patient will hear the tuning fork more loudly when it is touching the mastoid bone behind his affected ear than when it is held next to the ear (Rinne). For a normal ear, the reverse is true.

Burning the palate (with hot coffee) would activate which brainstem nucleus?

Spinal nucleus of CN V The spinal nucleus of CN V serves as the pain and temperature nucleus for the head/face, including the roof of the oral cavity. This nucleus is a long thin column of cells that extends from the pons (at attachment of CN V) through the medulla and into the upper cervical levels of the spinal cord). It receives primary pain signals from multiple cranial nerves including CN V (most importantly), and CN VII, IX & X (each of which innervates some skin or mucous membrane in the head/face). The mesencephalic nucleus of CN V relays proprioceptive information from the jaw muscles to the motor nucleus of CN V. The solitary nucleus processes visceral sensory information and taste. Edinger-Westphal is the nucleus containing preganglionic parasympathetic neurons whose axons travel in CN III to the sphincter pupillae and ciliary muscle of the eye.

The auditory part of the vestibulocochlear nerve is composed of axons of which of the following?

Spiral ganglion The spiral ganglion contains bipolar neurons. One process contacts a hair cell, the other makes up the cochlear part of CNVIII. Scarpa's ganglion contains the cell bodies of the vestibular afferents.The geniculate ganglion contains cell bodies of taste afferents from the anterior tongue and a few general sensory afferents from skin around the external ear. The nodose ganglion is a sensory ganglion of CN X, it contains the cell bodies of some visceral afferent fibers.

A patient with a facial nerve paralysis suffers from inability to dampen loud noises (hyperacusis) due to denervation of which muscle?

Stapedius Muscles attaching to the ossicles are believed to check excessive vibrations that could result in potentially damaging loud noises. If the muscles are paralyzed, increased ossicle movement results in louder sounds in the affected ear (hyperacusis). The stapedius is attached to the stapes and innervated by a branch of the seventh nerve (facial n); the tensor tympani is attached to the malleus and innervated by a branch of the trigeminal nerve.

A patient with a facial nerve (CN VII) paralysis suffers from hyperacusis due to denervation of which muscle?

Stapedius The facial nerve has several functions: sensory innervation of taste receptors in the anterior 2/3 of the tongue parasympathetic innervation of the lacrimal, submandibular and sublingual glands and glands of the mucosa in the nasal and oral cavities sensation to a small area of skin of the ear motor innervation to the muscles of facial expression and a couple of other small muscles derived from the second pharyngeal arch. Stapedius is one of these muscles. When CN VII is damaged, the denervation of stapedius allows greater movement of the ossicles and increased sound transmission in the affected ear (hyperacusis).

Degeneration of dopaminergic cell populations within what midbrain structure is associated with early stage Parkinson disease?

Substantia nigra Dopaminergic (DA) projections originate from cells in the substantia nigra (Latin for "dark substance", due to its dark appearance resulting from high levels of neuromelanin in dopaminergic neurons) in the midbrain. Loss of the dopaminergic cells is associated with Parkinson's disease. Therapies that restore DA levels can help with patients' symptoms, but cannot stop the continued degeneration of the DA cells.

Which of the following is the earliest level in the auditory pathway at which "BINAURAL" input to the same neuron occurs?

Superior olivary nucleus The superior olive computes the location of sound sources in a horizontal plane, by comparing differences in the volume and time of arrival of a sound at each ear; the cells in the superior olive are the first in the auditory pathway to receive input from both ears.

You design a research study to evaluate the diagnostic value of neuroimaging in early Alzheimer's disease. You plan to measure the width and thickness of the parahippocampal gyrus, which is affected early in the disease. Which imaging modality will you choose?

T1 MRI T1 MR shows anatomical detail of brain best.

The extent of tissue infarction by a recent (subacute) ischemic stroke would be best shown by this imaging method

T2 MRI

A lower motor neuron consists of:

The cell body located in the spinal cord or brain stem, and its axon projecting to the muscle. This is worth remembering. Damage to an LMN cell body in the CNS has the same result as damage to its axon in the PNS. So, it is not strictly accurate to equate LMN signs and symptoms with only PNS damage, although PNS damage can only produce LMN signs and symptoms. Remember that voluntary motor axons in cranial nerves have their cell bodies in the brainstem, and are analogous to the spinal LMNs. Recall this also: damage to the LMN cell body in the CNS, or damage to its axon close to the CNS generally means the neuron dies, while damage to the axon more distally may be repaired by regrowth in the PNS under the right conditions.

Which of the following is true of slow motor units?

They contract slowly and produce low forces A motor unit consists of a single motor neuron and its associated muscle fibers, while a motor neuron pool consists of all of the lower motor neurons (LMNs) responsible for activation of a given muscle, forming cell columns in the spinal cord. Both motor units and motor neurons vary in size, and larger motor neurons tend to innervate larger, more powerful motor units. There are three functional types of motor units: 1. Slow motor units - smaller, "red" fibers; contract slowly, and generate relatively low forces; resistant to fatigue due to high myoglobin content, large numbers of mitochondria, and dense capillary beds; important for activities that require sustained contraction. 2. Fast, fatiguable motor units - large, pale fibers that generate great force; sparse mitochondria; rapid rundown of response; brief, high force exertions, such as running or jumping 3. Fast, fatigue-resistant motor units - intermediate size, strength, and duration.

Notes about alar plate, neural crest and basal plate origin

The neural crest gives rise to a wide variety of cell types including ganglion cells in the PNS, Schwann cells, melanocytes, adrenal medulla, enteric nervous system, parts of the heart, cranium, face and neck. The alar plate gives rise to neurons that process sensory input (e.g., in the dorsal horn and sensory nuclei of the trigeminal nerve in the brainstem). The basal plate gives rise to motor neurons of the cord (ventral horn) and brainstem (motor neurons in cranial motor nuclei, for example the facial nucleus and the trigeminal motor nucleus)

A stroke resulting in visual defects could involve: (choose all that apply)

The posterior cerebral artery The ophthalmic artery The posterior cerebral artery supplies the primary visual cortex, which processes information from the contralateral visual field. The ophthalmic artery supplies the structures in the orbit, including the retina and optic nerve: damage to it will affect the vision in one eye.

The pigment epithelium is essential for maintaining the health and viability of photoreceptors in central retina.

The retinal pigment epithelium (RPE) is a single layer of post-mitotic cells (cuboidal epithelium), which functions both as a selective barrier to and a vegetative regulator of the overlying photoreceptor layer, thereby playing a key role in its maintenance. Through the expression and activity of specific proteins, it regulates the transport of nutrients and waste products to and from the retina, it contributes to outer segment renewal by ingesting and degrading the spent tips of photoreceptor outer segments, it protects the outer retina from excessive high-energy light and light-generated oxygen reactive species and maintains retinal homeostasis through the release of diffusible factors.

You are testing a patient's pupillary light reflex and observe the following situation. When a light is shone into the left eye, only a direct response is obtained. However, when a light is shone into the right eye, only a consensual response is obtained. This is an indication that:

There is a lesion of the right efferent pathway (CN III) The wording of the question tests whether you understand the meaning of the terms direct and consensual with regard to the pupillary light reflex, as well as understanding the neural basis of the reflex. In this patient, the right eye is non-responsive regardless of the eye that is stimulated with the flashlight, indicating a likely lesion in the right oculomotor nucleus or nerve. Checking and comparing each eye's light response helps to localize a lesion. In this case, because there is a light response in at least one eye with light shown in either eye, we conclude the afferent part of the reflex is intact (ie both optic nerves are functional). Because there is more than one route by which information from one eye can access the contralateral Edinger-Westphal nucleus (eg in the optic chiasm and in crossing fibers of the posterior commissure in the midbrain), lesions in any one of those routes do not produce an obvious defect in the response, so they can generally be dismissed as sites of lesion causing problems in the reflex.

A patient with an acute subdural hemorrhage (due to rupture of veins bridging from the cortical surface to the superior sagittal venous sinus) becomes restless and confused. Dilation of the ipsilateral pupil is noted on exam. The physician orders i.v. mannitol for which of the following reasons?

To promote osmotic diuresis to decrease ICP Damage to the brain (including any intracranial hemorrhage) will cause cerebral edema; edema and bleeding will increase the volume of intracranial contents and cause an elevation in ICP. If compensatory mechanisms are overwhelmed, there is a danger of brain herniation and death. Dilation of the pupil can signal uncal herniation which is compressing the oculomotor nerve (CN III). Mannitol will decrease cerebral edema: it promotes osmotic diuresis by increasing the pressure gradient, drawing fluid from intracellular to intravascular spaces. Although mannitol can be used to decrease intraocular pressure and increase blood pressure, the quick reduction of ICP in this patient is the key objective. Other treatments for raised ICP are elevating the head of the bed, intubation and hyperventilation to decrease pCO2 (causes vasoconstriction), draining CSF, craniotomy to evacuate a hemorrhage or provide decompression of intracranial cavity.

All cranial nerves innervate ipsilateral targets relative to the side where they emerge from the brainstem.

True All cranial nerves innervate ipsilateral targets relative to the side where they emerge from the brainstem.

Risk factors for disease progression in thyroid eye disease include:

Use of tobacco

Which of the following pairs of cranial nerves contain the afferent and efferent limbs respectively of the corneal blink reflex?

V and VII

What shows up white in T1 MRI?

White matter T1 MRI is a good imaging modality for assessing morphological detail of brain. White matter appears pale and gray matter appears darker; CSF appears black. Bones are not seen with T1 MRI.

Which of the following is not a physiologic role of astrocytes?

Wrap axons with myelin Astrocytes are star shaped glia cells that provide many support functions for the CNS. However, they do not produce myelin, which is provided by oligodendrocytes in the CNS, and Schwann cells in the PNS.

GABA is NOT

able to activate its receptors to increase sodium ion conductance Gamma Amino Butyric Acid (GABA), a simple amino acid, is the major inhibitory CNS transmitter, distributed ubiquitously throughout the brain and spinal cord, with higher concentrations in diencephalic areas and lower amounts in cerebral cortex, cerebellar cortex and lower brain stem. Two classes of GABA receptors have been identified: GABAA and GABAB receptors · GABA-A receptors are ionotropic and regulate neuronal excitability by increasing Cl- conductance. · GABA-B receptors are metabotropic (GI/O coupled) and regulate neuronal excitability by both increasing K+ conductance and decreasing Ca++ conductance. GABA inhibits neurons both postsynaptically and presynaptically. While both receptor subtypes are expressed pre- and post-synaptically, GABA-A receptors play a dominant role in postsynaptic inhibition whileGABA-B receptors play a dominant role in presynaptic inhibition. The actions of several classes of psychoactive drugs, including benzodiazepines, barbiturates, and ethanol, are mediated via GABA-A mechanisms. The natural plant compound, picrotoxin, long known as an analeptic-convulsant agent, is a noncompetitive GABAA receptor antagonist that blocks the chloride channel of the receptor. Another plant-derived poison that produces seizures is bicuculline. It is a competitive antagonist at GABA-A receptors. The drug, baclofen, known as a muscle relaxant (spinal cord depressant), is a GABA-B agonist, mimicking the actions of GABA at the GABA-B receptor.

Nicotine is an agonist that activates

acetylcholine receptors ACh acts as both a fast and a slow neurotransmitter via two classes of receptors, originally identified pharmacologically: · Muscarinic (muscarine-activated) receptors are metabotropic and act through either Gi/o or Gq G-proteins. Muscarinic receptors are often autoreceptors on the presynaptic terminal that regulate subsequent acetylcholine release. · Nicotinic (nicotine-activated) receptors are ionotropic, ligand-gated channels permeable to Na+ and Ca++. Peripheral and central nicotinic receptors have different pentameric subunit composition and consequently have somewhat different properties. The nicotinic partial-agonist succinylcholine is used as a paralytic (neuromuscular blocker) during surgery. Varenicline is a nicotinic partial agonist used as a smoking cessation aid.

Pathways from the vestibular nuclei to the spinal cord

activate neck muscles to keep the head upright. The vestibular nuclei are the origin of the vestibulospinal tracts, which activate medially located motor neurons in the spinal cord ventral horn. These tracts: help maintain upright posture - an essential function because we stand and walk balanced on very small supports, the soles of our feet. Without continuous (though unconscious) vestibular or other sensory feedback, you would tip over any time you tried to stand. activate neck muscles to keep the head upright. allow for head and limb extension when balance is threatened.

Which of the following is responsible for return of CSF into the blood stream (dural venous sinuses)?

arachnoid granulations (villi)

In the primary visual cortex of the human, the representation of the most central region of the visual field (macula or fovea) is located:

at the most posterior pole of the occipital lobe The retinotopic map in primary visual cortex is orderly--upper field is represented on lower bank of calcarine cortex, lower field on upper bank, peripheral field more anterior along the calcarine sulcus, and central (foveal) field towards the occipital pole (most posterior part of the occipital lobe). The retinotopic map is distorted in the primary visual cortex: the central field representation is disproportionately large because much more cortex is needed to process information from the high density of photoreceptors in the fovea.

NMDA receptors do NOT

bind Beta-carbolines as inverse agonists to decrease chloride conductance Glutamate is the most abundant excitatory amino acid transmitter in CNS and in peripheral autonomic and somatic nervous systems. Glutaminergic transmission underlies normal behavioral states and memory formation. Drugs that increase release of glutamate or mimic actions of glutamate can produce seizures, and with excess amounts, neurotoxicity and death of neurons. Ionotropic glutamate receptors are pentameric ionophores of 2 functional classes, named for the prototypical agents that affect them selectively: · AMPA/Kainate class selectively passes Na+. · NMDA class will conduct both Ca++ and Na+. NMDA receptors' Ca++ conduction is important in the synaptic strengthening that is thought to underlie learning and memory. Hyperstimulation of NMDA receptors is excitotoxic because excess Ca++ entry leads to apoptosis and neurodegeneration.

At typical doses in caffeinated beverages, caffeine produces arousal by

blocking adenosine A2 receptors Adenosine is a purine nucleoside with a wide range of physiological functions including neurotransmission. Adenosine receptors are G-protein coupled receptors that either inhibit cells by activating Gi/o signaling (A1-type), stimulate cells by activating Gs signaling (A2- type), or stimulate phospholipase activity by activating Gq signaling. In general, adenosine has an inhibitory effect in the central nervous system. Caffeine is a CNS stimulant because it acts as an antagonist of adenosine receptors (A1 and A2A types). At higher doses, caffeine is also a phosphodiesterase inhibitor (hydrolysis of cAMP to AMP is blocked). And at still higher concentrations, caffeine blocks Ca++ release from ryanodine-sensitive cytoplasmic stores. At typical doses in caffeinated beverages, the net result is arousal caused by antagonism of A2A receptors and a subsequent increase in NE activity in brain.

Which of the following represents a serious contraindication for MRI studies?

cardiac pacemaker

A patient has weakness or paralysis on one side of the body, and reduced or absent eye movements toward that same side. Typically this indicates a lesion of the

cerebral cortex including the frontal eye field. If a patient has weakness or paralysis on one side of their body, and reduced or absent eye movements toward that same side, the lesion in this case is cortical, affecting the motor cortex and frontal eye field, or their descending tracts. Recall that the frontal eye field on one side controls saccadic eye movements to the opposite side. So, the intact frontal eye field will be unopposed, and at rest, the patient's gaze will tend to be directed toward the side of the lesion. In this clinical presentation, they eye movement abnormality is referred to as "right way eyes".

Most of the cerebrospinal fluid is produced in the

choroid plexus

A majority of large off-midline arterial vessels that supply the brainstem contain penetrating branches that encircle the brainstem. These secondary branching vessels that innervate the dorsolateraral surface and structures of the brainstem are referred to as the:

circumferential branches All of the large, off-midline arteries of the brainstem, (e.g. posterior inferior cerebellar artery, PICA) have a distribution of small vessel branches that encircle the brainstem and give off branches that penetrate the brainstem parenchyma. Along the dorsolateral surface these branches are referred to as the circumferential branches. As a general rule of thumb for the brain, an artery on the surface of the brain supplies the underlying tissue. Rather than, say, learning detailed maps of arterial territories for each brainstem level in cross-section, it may be worthwhile being able to draw the location of the major vessels on the ventral view of the brainstem. That way, you can infer the supply of the underlying tissue.

Referred pain is most likely to occur because of:

convergence of somatic and visceral C fibers on the same dorsal horn cells. Referred pain is the phenomenon in which pain originating in one structure is mistakenly perceived as originating from another structure. The mechanism of referred pain is not well understood, but is believed to be due to some convergence in the spinal cord, where nociceptive afferents from two sites converge on the same neuron. This explanation does not account for the observation that one of the afferents is often "dominant" eg pain from the heart and from parts of the arm are believed to converge, but pain from the arm is not referred to the heart. Nevertheless, patterns of referred pain are important clinically and you should consider referred pain as a differential when a patient presents with pain.

In which part of the inner ear are the hair cells associated with the barrel-shaped cupula, which detect angular acceleration and circular rotation of the head?

cristae ampullae

Which finding would NOT be present in a patient with a thoracic spinal cord hemisection on the left side?

decreased position sense and tactile discrimination in right leg. Brown-Sequard or cord hemisection disrupts 1. ascending primary afferents from below the lesion--causing loss of discriminative touch, proprioception, vibration and pressure ipsilaterally 2. second order axons in the nociceptive pathway (ie spinothalamic tract) which have crossed the midline--causing loss of pain and temperature sensation contralateral to the lesion There would also be interruption of the descding motor pathways causing weakness on the same side as the lesion. Because the lesion is in the thoracic cord, the upper limb is spared.

On a cross section through the brain the gray matter on either side of the third ventricle is the

diencephalon Another good landmark for recognizing deep brain structures: the 3rd ventricle is a midline slit; its upper walls are formed by the thalamus on each side, while the hypothalamus lies in its lower walls and forms the floor of the ventricle. Thalamus and hypothalamus together make up the diencephalon (along with the retina and optic nerve).

Substantia nigra neurotransmitter

dopamine

Ventral tegmental area neurotransmitter

dopamine

An aneurysm of the posterior communicating artery is most likely to cause which of the following?

enlarged pupil The posterior communicating and posterior cerebral arteries lie close to CN III, the oculomotor nerve. Aneurysms often form on these arteries, close to their junction, and when the aneurysm enlarges, it can compress CN III. The parasympathetic fibers in CN III lie on the surface of the nerve and are thus the first to be affected by compression, leaving the sympathetic fibers unopposed in their action, causing the pupil to dilate. As the other motor fibers are affected in CN III, ocular movement can be impaired and the eyelid may droop as CN III innervates the levator palpebrae superioris.

A lesion of the right medial lemniscus in the upper medulla will result in the loss of:

fine touch and proprioception from the left body only Two things to consider in this question: the tract that is lesioned and the point where it is lesioned. The medial lemniscus is formed by the axons of second order neurons in the discriminative touch, proprioception, vibration and pressure pathway, once they cross the midline in the medulla. So, the patient will experience a loss of conscious awareness of these sensory modalities from the opposite side of the body to the lesion. As the ML ascends to the thalamus, second order fibers from the main sensory nucleus of the trigeminal add to it at the level of the upper pons (though sometimes these are given their own name, the trigeminal lemniscus). So, damage to the medial lemniscus at rostral pons level or above may also lead to similar loss in the face/head opposite the lesion.

This contrast agent is useful in some circumstances for enhancing the appearance of tumors shown by T1 MRI.

gadolinium Clinical MRI scans often use an IV contrast agent containing gadolinium, which shortens T1 relaxation time and improves diagnostic accuracy, by increasing the signal to noise within an image. Because gadolinium does not normally cross the blood-brain barrier, its presence in brain parenchyma indicates a compromised blood-brain barrier e.g. in stroke and in tumor proliferation.

Pathologically increased intraocular pressure is called glaucoma

glaucoma Aqueous humor is secreted by the ciliary body and flows through the pupil into the anterior chamber. Open-angle glaucoma is a condition in which the drainage of aqueous humor into the collecting vein (canal of Schlemm) in the anterior chamber lags behind the rate of secretion. Accumulating aqueous humor causes pressure to build up inside the eye and damage axons of retinal ganglion cells. Because severed axons in the CNS do not regenerate, over time the optic nerve shrinks. Visual deficits are first evident in peripheral visual field, suggesting that axons coming from the retinal periphery die first. Open angle glaucoma represents around 90% of all cases of glaucoma, and afflicts about 3 million Americans.

Ultrasound has limited application for brain imaging because of the

high acoustical impedance of the skull Ultrasound is not capable of resolving tissue types with high acoustical impedance (e.g. bone, air) and is thus limited in evaluating structures encased in bone (e.g. brain tissue) without an access window. The most common diagnostic application of ultrasound is the use of doppler ultrasound to assess vascular integrity and flow parameters. Doppler ultrasound detects a frequency shift in an echo occurring at a given location that is a function of blood flow through a vessel. This shift is direction- and speed-dependent, based on the Doppler effect and can be used to determine lumen diameter of large vessels and direction of vessel blood flow (i.e. moving toward or away from the transducer probe). This approach in the neck is helpful to assess carotid stenosis. Transcranial Doppler approaches have also been developed. Although the skull bone blocks transmission of ultrasound, regions with thinner walls can be used as access ports. For example, in the temporal bone region above the zygomatic arch transcranial Doppler ultrasound can be used to evaluate flow in proximal branches of various cerebral arteries. This technique is used in intensive care units to detect vasospasm following subarachnoid hemorrhage.

Which of the following is a prosencephalic but not a telencephalic derivative?

hypothalamus Prosencephalon gives rise to the telencephalon and diencephalon Telencephalon derivatives: the cerebral cortex (including hippocampal formation), amygdala, and the basal ganglia (caudate, putamen and globus pallidus) Diencephalic derivatives: thalamus, hypothalamus, retina and optic nerve

Retinal ganglion cells in the right retina send axons to the right thalamus (specifically the lateral geniculate nucleus):

if they are located in the temporal half retina Temporal retinal ganglion cell axons continue uncrossed through the optic chiasm. Axons of nasal retinal ganglion cells cross at the chiasm.

Low midbrain

image

Pons

image

Rostral or open medulla

image

Damage to the nucleus ambiguus, a cranial nerve nucleus would be closely associated with:

impaired coughing, swallowing, & phonation The nucleus ambiguus is the motor nucleus for muscles of the pharynx and larynx. Named ambiguus (literally means ambiguus) due to the fact that it is difficult to see in conventionally stained material, this nucleus contains cell bodies of lower motor neurons of CN IX & X that project to muscles of the pharynx, larynx and soft palate. Thus, the nucleus ambiguus coordinates pharyngeal functions such as coughing, swallowing, & phonation. It also serves as the nucleus involved in the motor component of the gag reflex.

A young adult was found unconscious, apparently due to a drug overdose. His axial FLAIR image shows multiple lesions in a linear parasagittal location in both hemispheres, ascribed to prolonged hypotension. What is the diagnosis?

internal (deep) watershed infarct Watershed cerebral infarctions, also known as border zone infarcts, occur at the border between cerebral vascular territories where the tissue is furthest from arterial supply and thus most vulnerable to reductions in perfusion. Watershed cerebral infarction accounts for 5-10% of all cerebral infarctions. They tend to be seen in the elderly, who have a higher incidence of arterial stenosis and hypotensive episodes, as well as microemboli. Episodes of systemic hypotension particularly with severe stenosis or occlusion of the feeding arteries, in particular intra and extracranial carotid arteries, is the typical scenario in which watershed infarction is encountered, especially bilaterally. In this case, a drug overdose caused a prolonged period of hypotension. 5B syndrome is fictitious!

A molecule of CSF travels from the inferior horn of the lateral ventricle to the fourth ventricle. It must pass through the

interventricular foramen (of Monro) and cerebral aqueduct (of Sylvius)

For the eye, choose the correct association:

iris: has a smooth muscle that dilates pupil The iris is composed largely of 2 smooth muscles that either dilate (dilator pupillae) and constrict (sphincter pupillae) the pupil under the control of the sympathetic and parasympathetic systems respectively. The lens separates the anterior and posterior cavities of the eye: the posterior cavity contains the vitreous (a gel-like substance present from birth), while the anterior cavity is divided by the iris into anterior and posterior chambers containing aqueous humor. Cataract is a pathology affecting the lens; glaucoma is a pathology affecting the drainage of aqueous humor (among other factors), associated with increased intraocular pressure.

A patient has been diagnosed with a tumor involving cranial nerve VII on the right side. Which response would you expect when the right cornea is touched with a cotton applicator?

left eye blinks The corneal blink reflex occurs when, in response to the cornea of one eye being gently touched with a sterile cotton applicator, both eyes blink. The sensory limb of the reflex is a branch of CN V1 (ophthalmic nerve) on the side of the stimulus, and the motor limb of the reflex is CN VII (facial nerve) bilaterally. Thus the reflex is a quick test of CN V on the side of the stimulus, CN VII on both sides, and the intervening brainstem circuits (in the pons and medulla) connecting the two. Both corneas should be tested. If CN VII is damaged on one side, no blinking will occur on that side regardless of which cornea is stimulated. If CN V1 is damaged on one side, neither eye will blink when that cornea on the side of the damage is stimulated, but both eyes will blink when the cornea on the intact side is stimulated.

A disease that often affects older people is age-related macular degeneration (AMD), in which the macula lutea deteriorates progressively. Deduce the symptoms of AMD.

loss of vision in the center of one's visual field The macula lutea (yellow spot) is adjacent to the fovea, the region of the retina responsible for high acuity vision in the center of the visual field. Although the term macula is also used to describe parts of the utricle and saccule, indicating a high density "spot" of receptors, AMD refers specifically to the degeneration of the macula lutea.

Otoliths (otoconia) in the ear are associated with

maculae of the utricle and saccule Normally, otoconia (otoliths) are found in the utricle and saccule. If otoliths become dislodged they can float around and cause abnormal fluid movement resulting in vertigo.

The cerebral aqueduct (aqueduct of Sylvius) passes through the part of the brain derived from the:

mesencephalon General answer comments The lumen of the neural tube becomes distorted during development, eventually forming the ventricular system in each part of the brain: telencephalon (cortex and basal ganglia): lateral ventricles (one in each hemisphere) diencephalon (thalamus and hypothalamus): third ventricle mesencephalon (midbrain): cerebral aqueduct metencephalon (pons and cerebellum): upper part of the fourth ventricle myelencephalon (medulla): lower part of the fourth ventricle and central canal spinal cord: central canal (not patent below cervical levels) Note: the metencephalon and myelencephalon are derivatives of the rhombencephalon (hindbrain)

Branches of which of the following vascularize the primary auditory cortex, Wernicke's area, and the sensorimotor cortex of the face?

middle cerebral artery

Blockade of this receptor will cause a marked fall in blood pressure.

nicotinic receptors in autonomic ganglia

Locus ceruleus neurotransmitter

norepinephrine

Multiple sclerosis is a demyelinating disease of the central nervous system. Which glial cell is affected by MS?

oligodendrocytes Recall that oligodendrocytes myelinate axons in the CNS while Schwann cells myelinate axons in the PNS. MS affects oligodendrocytes and impacts the myelin in the CNS, while Guillain-Barre attacks peripheral myelin.

A patient with monocular blindness would most likely have a damaged:

optic nerve Monocular blindness (or scotomas) generally result from pre-chiasmatic lesions of the ipsilateral eye or optic nerve. Information from both eyes is represented in the post-chiasmatic visual pathway, so damage to the optic tracts and the continuation of the visual pathway to the cortex results in hemianopia and loss of some part of each eye's field.

BPPV (benign positional paroxysmal vertigo) is caused by

otoconia that become detached from otolithic membranes. Otoconia from the utricle or saccule become detached and float around in the endolymph. When they get into one of the semicircular canals they can cause movement of the endolymph, which is detected by hair cells in the canal and reported to the brain as head rotation. Often vertigo caused by BPPV occurs when the head moves into a particular position, and can be alleviated by shifting the head to a different position.

A growing intracranial mass may cause the medial temporal lobe to herniate

over the edge of the tentorium cerebelli, compressing CN III. The medial temporal lobe lies on the tentorium close to the tentorial notch; therefore with increased ICP due to a supratentorial mass, it can herniate through the tentorial notch compressing the midbrain. Transtentorial herniation is the most common type of herniation. It results from downward displacement of supratentorial brain tissue (uncus or diencephalon) into the infratentorial compartment and can be caused by supratentorial mass lesions, diffuse brain swelling, focal edema, or acute hydrocephalus. Transtentorial herniation can cause compression of the third cranial nerve, and the midbrain, (especially the cerebral peduncles) as well as distortion or traction of the superior portion of the basilar artery and the posterior cerebral arteries, leading to occipital lobe infarction. Subfalcine herniation occurs when increased pressure in one hemisphere displaces brain tissue under the falx cerebri; it can cause compression of the anterior cerebral artery and extensive infarction of the frontal and parietal lobes, including the sensorimotor cortex for the lower limb. Foramen magnum herniation occurs when downward pressure forces the cerebellar tonsils into the foramen magnum, where they compress the medulla oblongata and vertebral arteries.

The spinothalamic pathway carries information about

pain (nociceptive information) Two key somatosensory pathways to remember forever are the 1. Dorsal column/medial lemniscus pathway that carries discriminative touch, proprioceptive, vibration, pressure information for conscious awareness 2. Spinothalamic (anterolateral) pathway that carries nociceptive (pain) information, as well as temperature, crude (poorly localized) touch, itch, and tickle for conscious awareness.

The gray ramus communicans is a branch from the sympathetic chain ganglion neuron, that travels through ________________________ nerves/ to reach __________________________.

peripheral nerves/ sweat glands, arrector pilae, smooth muscle of blood vessels Blockade of ganglionic nicotinic receptors will cause a marked fall in blood pressure, eg. mecamylamine.

Most of the blood supply to the visual cortex is via which artery?

posterior cerebral The PCA arises from the basilar artery and runs immediately superior to the tentorium around the cerebral peduncle to supply the inferior surface of the temporal lobe and the medial surface of the occipital lobe-thus supplying the primary visual cortex around the calcarine sulcus. The most posterior pole of the occipital lobe may get some of its blood supply from the MCA--and this could account for the macular sparing sometimes seen with a PCA blockage.

The cortex on the medial surface of the brain, posterior to the parieto-occipital sulcus, is supplied by the

posterior cerebral artery The medial surface of the hemisphere has two arteries supplying it: anterior cerebral artery and the posterior cerebral artery. The territories are separated by the parieto-occipital sulcus. note: THERE IS NO MEDIAL CEREBRAL ARTERY -- so forget you ever heard this term!! The MIDDLE cerebral artery supplies most of lateral surface of the hemisphere: close to the Circle of Willis the middle cerebral artery is more or less "in the middle" between the proximal parts of the anterior and posterior cerebral arteries.

The vestibulo-ocular reflex coordinates head and eye movement. Which of the following statements about the VOR is true? Select all that apply.

relies on the integrity of semicircular canals, vestibular nuclei, ascending medial longitundinal fasciculus, and oculomotor nuclei it allows you to read text when you wiggle your head back and forth it allows you to read text when you wiggle your head back and forth is tested in the doll's eyes maneuver Vestibular input to extraocular motor nuclei mediates the VOR, a reflex that is essential for clear vision whenever you are moving. When your head rotates (for example, bounces slightly up or down during walking), your eyes rotate in the opposite direction by an identical amount, so that the image of the visual scene remains stationary on your retinas. The VOR is extremely fast, and happens without conscious awareness.

Cones are:

responsible for high acuity vision Cones are very densely packed into the fovea, the small depression at the optical center of the retina. Their density declines precipitously as one moves away from the fovea. Cones provide high acuity vision, so this very uneven cone distribution means that we have excellent acuity only in central retina, particularly the fovea, which is less than 2 deg in diameter. For this reason whenever we want to see fine detail, for example when reading, we direct our center of gaze, the fovea, to the target. A given cone contains either "red", "green", or "blue" photopigment, depending on its wavelength sensitivity. In people with normal color vision, red cones are most numerous, followed by green cones, with blue cones being relatively sparse. Note that each photopigment responds to a range of wavelengths centered around its optimal wavelength - thus when we see, for example, a particular shade of yellow, both our red and green cones are active, with a unique balance between the levels of activity in these two populations that signals this particular yellow.

A previously healthy 31-year-old individual experiences a sudden severe headache and photophobia. An emergent head CT is unrevealing. The patient is otherwise healthy, has no prior history of headache, and is afebrile. A lumbar puncture yields cerebrospinal fluid with many red blood cells, but no white blood cells. The CSF protein is slightly increased, but the glucose is normal. Which of the following is the most likely cause?

ruptured berry aneurysm About 1% of the population have berry aneurysms. Berry aneurysms are most likely to involve the Circle of Willis, so that rupture with bleeding occurs into the subarachnoid space at the base of the brain. The blood may cause irritation and spasm of nearby arteries causing worsening of symptoms from ischemia. Berry aneurysms arise from a weakness in the cerebral arterial wall present from birth; they can slowly enlarge over many years, but they can rupture suddenly. The LP did not reveal WBC's, and she was afebrile, ruling out bacterial meningitis. Hypertension leads to arteriolosclerosis with intraparenchymal hemorrhage, but this patient is a healthy young adult. Cerebral amyloid arteriopathy involves small peripheral arteries at the neocortical surfaces. Cerebral amyloid arteriopathy is most likely to occur in association with Alzheimer disease, thus unlikely in a 31yr old.

Serotonin syndrome can be caused by toxic doses of

selective serotonin reuptake inhibitors Serotonin syndrome (SS) is a potentially life-threatening reaction to drugs that elevate synaptic serotonin levels in brain and PNS. Excess serotonergic activity causes cognitive, autonomic and somatic effects. Mild symptoms include increased heart rate, shivering, sweating, dilated pupils, myoclonus (intermittent tremor or muscle twitching), and hyperreflexia. At higher doses, hyperactive bowel sounds, elevated blood pressure and hyperthermia may be evident. Mental status changes may include confusion, agitation, hypomania, hallucinations and hypervigilance. Life-threatening effects include shock, hyperthermia, metabolic acidosis, and seizures. Numerous psychotropic drugs and combinations have the potential to produce SS. These include direct 5HT-receptor agonists, serotonin reuptake inhibitors, and monoamine oxidase inhibitors used in the treatment of psychosis and depression. Some opioid analgesics (tramadol & methadone), some psychotomimetics (MDMA & LSD), some herbs (St John's wort) and some dietary supplements (tryptophan) may produce SS. Usually, a careful patient history can identify the cause, but the symptoms can often be confused with neuroleptic malignant syndrome (NMS). The principal difference between SS and NMS is that NMS is caused by excessive DA receptor blockade rather than hyperserotonergic stimulation. Treatment of SS includes reducing the causative agent, administration of serotonergic antagonists, administration of benzodiazepines (to reduce seizures, tremors, or agitation).

Migraine pain can be effectively treated using

serotonergic 5HT1B/D receptor selective agonists (triptans)

raphe nuclei neurotransmitter

serotonin

A 16 year old girl is involved in a motor cycle accident; she was not wearing a helmet. On examination, she is comatose, has bruising around the eyes and behind the right ear, and drainage of clear fluid from the nose and right ear. Based on these findings, which of the following is most likely present in this case?

skull base fracture Signs of skull fracture include periorbital ecchymoses or hematoma (racoon eyes), postauricular ecchymoses (Battle's sign), CSF rhinorrhea, and otorrhea. Given the clinical findings, skull fracture is most likely to be present in this case, although the patient's injuries may also include other injuries associated with head trauma, such as scalp injury, cervical injury, skull fracture, cerebral contusion, and epidural, subdural, intraparenchymal and/or subarachnoid hemorrhage.

The most likely localization of a lesion in a patient with sensory loss of the bilateral legs and trunk below T10 is:

spinal cord

The first synapse in the pathway for pain and temperature for the face is located in the

spinal trigeminal nucleus The pain pathway from the face is unusual in that once the primary afferents (with their cell bodies in the trigeminal ganglion) reach the CNS, they turn caudally to reach their target nucleus, the spinal trigeminal nucleus. After synapsing there, the information flows rostrally along the axons of the spinal trigeminal neurons to the contralateral VPM nucleus of the thalamus.

Choose the correct sequence in the auditory pathway of sound vibrations and nervous impulses. All these choices tell just a part of the pathway (none is complete), but only one choice has no errors.

stapes moves the oval window, which causes pressure waves that vibrate basilar membrane and deflect stereocilia of hair cells embedded in the tectorial membrane. Conduction of sound waves to the auditory hair cells involves in order: vibration of the tympanic membrane and ossicles, resulting in pressure transmitted by the stapes to the oval window and movement of the fluid in the cochlea, which causes movement of the basilar membrane and activation of the hair cells as their stereocilia are deflected towards or away from the kiniocilium.

With age, the lens becomes

stiffer Starting in middle age, the lens begins to lose its elasticity, and it cannot round up as much during accommodation as can a normal lens; this condition is presbyopia. Corrective lenses can improve vision. If the lens loses its transparency (cataract) it must be removed to restore/improve vision.

Venous blood from only deep brain structures drains via the:

straight sinus Deep brain structures drain to the great cerebral vein of Galen which is the main tributary of the straight sinus along with the inferior sagittal sinus at the inferior edge of the falx cerebri. Superficial veins on the cortical surface drain mostly to the superior sagittal sinus.

The great cerebral vein drains into the

straight sinus The great cerebral vein (of Galen) is a single, midline vein that drains deep structures of the brain. It joins the inferior sagittal venous sinus to form the straight sinus.

This/these structure(s) can tear in a head injury leading to a subdural hematoma:

superficial ("bridging") veins Superficial veins over the surface of the cortex drain into the superior dural sagittal venous sinus. They are vulnerable to shear injury as they cross ("bridge") from the arachnoid into the dura. Venous blood spreads ("dissects") relatively easily between the dura and arachnoid, spreading over a large area and forming a crescent-shaped hematoma. Chronic subdural hematoma is seen in the elderly after minor or sometimes no history of trauma: with age, the brain atrophies, increasing the stretch on the bridging veins, as the brain moves more freely in the cranial cavity. Acute subdural hematoma, in contrast, occurs after high velocity impact head injury.

A patient displays a symmetrical loss of pain and temperature across the shoulder area on both sides of the body (ie cape-like loss of pain and temperature sensation) with no loss of tactile sensation. Pain and temperature and tactile sensation are normal over the rest of the body. This condition would be due mostly likely to:

syringomyelia Syringomyelia (an example of central cord syndrome) is due to a focal disruption in the central part of the cord, affecting the crossing pain pathway from each side of the body. At each level of the cord, fibers that carry pain information from the body dermatomes supplied by spinal nerves just below the level, cross the midline close to the central canal of the cord to reach the spinothalamic tracts. A single midline lesion damages just the crossing fibers resulting in bilateral dermatomal loss of pain and temperature. If the lesion is in the low cervical cord, it causes bilateral loss from dermatomes in the arms and shoulders where a cape would rest, hence the term "cape-like loss".

The most likely place for a lesion causing a quadrantic visual field scotoma is in the:

temporal portion (Meyer's loop) of optic radiation This is something that needs to be learned since it is not easily deducible from diagrams etc. From the LGN, axons carrying upper field information loop anteriorly into the temporal lobe before turning posteriorly and ending in the lower bank of the calcarine cortex. Thus damage to this loop--Meyer's loop--or to its termination in the occipital lobe could cause contralateral superior (or upper) quadrantanopia. Axons carrying lower field information project directly posteriorly to the upper bank of the calcarine cortex: damage to these axons or their terminals in the occipital lobe results in contralateral inferior (lower) quadrantanopia.

If a patient has a tumor affecting the left occipital cortex superior to the calcarine sulcus, you would expect to find a scotoma in:

the right inferior visual quadrants of both eyes' fields

You are assisting with caloric testing in a patient whose head cannot be rotated safely due to concerns about neck injury. Cold water is injected into the left auditory canal. The client exhibits conjugate eye movements toward the left followed by a rapid nystagmus toward the right. When warm water is introduced into the same ear, the nystagmus reverses. It is fair to say that

the vestibular nuclei are functioning Checking cranial nerve reflexes provides a quick crude assessment of the integrity of the nerves and the parts of the brain/brainstem through which they connect with each other. Caloric testing assesses the vestibular apparatus, CN 8, and connections between vestibular nuclei and extraocular nuclei in the pons and midbrain, and cranial nerve connections to extraocular muscles. The COWS mnemonic reminds us that if the vestibular system and its connections through the brainstem to the oculomotor nuclei are intact, then cold water causes nystagmus to the side opposite the cold water injection. No movement of the eyes indicates damage to the vestibular nuclei, oculomotor nuclei or their connections in the brainstem. If the reflex is abnormal further testing needs to be done to isolate the cause of the abnormality (ie in vestibular system, ocular motor system or connections between); eg., eye movements could be tested independent of the reflex if the patient can follow instructions.

A pituitary tumor that exerts pressure on the optic chiasm will often lesion the decussating (crossing) optic nerve fibers. In such a case, which of the following is true?

there will be a visual field loss in the temporal visual fields of both eyes (bitemporal hemianopia). Crossing fibers at the optic chiasm are from the nasal retinae, which because of lens refraction, "see" the temporal fields of each eye. Damage (eg due to a pituitary tumor compressing the axons at the chiasm) causes bitemporal hemianopia.

A 37 yr old woman presents to her family physician with intermittent physical problems. She is referred to a neurologist, and after a thorough examination a tentative diagnosis of multiple sclerosis is made. Imaging reveals an area of demyelination in the region highlighted in red and arrowed. Which of the following deficits would correlate most specifically with the location of this lesion?

tongue deviates to the right on protrusion, weakness in left upper and lower limbs. The lesion is at the level of the rostral medulla on the patient's right encompassing the right pyramid and a small region just lateral to it, where CN XII axon fascicles run to exit the brainstem between the pyramid and the olive. Hence, you would expect the deficits to be motor in nature: damage to CN XII on the right would cause weakness (LMN type) in the right tongue, so the tongue would deviate to the right on protrusion. Damage to the right pyramid interrupts the corticospinal tract before decussation, so you would expect UMN type weakness in the left upper and lower limbs.

Name the structure

transverse sinus

The largest numbers of fibers in the white matter columns of the cord are found at which cord level?

upper cervical Ascending somatosensory tracts have accumulated at the top of the cervical cord from all the lower levels of the body as they then continue en route to the forebrain. Descending tracts from the forebrain and brainstem that control motor function of the body are largest in the cervical cord and decrease in size along the length of the cord as descending axons leave the tract to terminate on their target motor neurons.

Somatosensory pathways from the body and head project to which thalamic nuclei?

ventral posterior The specific thalamic relay nuclei for somatosensory information are located in the ventral posterior part of the thalamus. VPL relays information from the body to primary somatosensory cortex; while VPM relays information from the head/face to primary somatosensory cortex. An additional nucleus (you probably won't see this in first aid books though) that processes nociceptive somatosensory information is the ventromedial nucleus, posterior part (VMpo), which projects to the insula.

A 71 yr old man is brought to the ER after collapsing while working in his garden. The man's partner reports they found him after he failed to come in to the house for lunch. He is conscious and coherent, and says his collapse "came on real sudden". Examination reveals a profound weakness of left upper and lower extremities, dilation of the right pupil, and loss of voluntary movement of his right eye. MRI shows a lesion of vascular origin. A thorough neurological examination would most likely reveal which of the following deficits in this man?

weakness of the left lower face The lesion can be localized to the right midbrain because of the involvement of CN III accompanied by left sided hemi-paresis. The lesion most likely involves the fascicles of CN III before their point of exit from the anterior midbrain, and the adjacent cerebral peduncle, where the corticospinal and corticobulbar (AKA corticonuclear) tracts are located. In addition to the limb weakness, this patient will show weakness in the lower face on the left, but will be able to wrinkle his forehead on both the left and right, because the upper face motorneurons in the facial nucleus (eg supplying frontalis) receive corticobulbar innervation from both sides of the cortex. The motor neurons for the lower face (ie around the mouth) are like limb motor neurons in receiving innervation from the only the contralateral cortex. Because of the trajectory of the corticobulbar fibers, this patient would also likely have tongue deviation to the left on protrusion because the hypoglossal motor neurons in the left hypoglossal nucleus would have lost their cortical innervation. Additionally, when asked to say "AH" the patient's uvula would likely deviate to the right, due to loss of cortical drive to the left nucleus ambiguus.


संबंधित स्टडी सेट्स

The Reproductive and Genitourinary Systems Pharm

View Set

Final Review for Accounting (Start with Definition)

View Set

Programming Fundamentals I Exam 3

View Set

CULTURA SPANISH MIDTERM, spanish midterm

View Set

MA RMV Practice Test 2 (my copy of notes)

View Set

Adult Medical Surgical Practice A

View Set

AP Euro Project Quarter 3 Quizlet #1

View Set

BIBL 104-Quiz: The Old Testament Books of Prophecy

View Set